8th

Ace your homework & exams now with Quizwiz!

In the short run A) the interest rate can rise when the domestic money supply falls. B) the interest rate can decrease when the domestic money supply falls. C) the interest rate stays constant when the domestic money supply falls. D) the interest rate rises in the same proportion as the domestic money supply falls. E) the interest rate never rises when the domestic money supply falls.

A

Is it possible for a country to have a current account deficit at the same time it has surplus in its balance of​ payments? A. Yes. B. No.

A

The prohibitive tariff is a tariff that A) is so high that it eliminates imports. B) is so high that it causes undue harm to trade-partner economies. C) is so high that it causes undue harm to import competing sectors. D) is so low that the government prohibits its use since it would lose an important revenue source. E) is so low that it causes domestic producers to leave the industry.

A

The reason Airbus succeeded in the Brander Spencer example is that A) the European government made an explicit subsidy offer, but the U.S. government did not. B) Airbus' prices were better when adjusted for quality and warranty services. C) Boeing traditionally refused to undertake any exchange rate risk in its transactions. D) the U.S. acted in accordance with its ideological reliance on market solutions, whereas the Europeans ignored market and technological factors. E) the Airbus plane benefited from more advanced technology.

A

The relationship showing that damage to the environment increases as a​ country's per capita income rises from an initially low level but then falls with further increases in its per capita income is known as the environmental A. Kuznets curve B. income cycle C. Engel curve D. Gore curve.

A

The strongest political pressure for a trade policy that results in higher protectionism comes from A) domestic workers lobbying for import restrictions. B) domestic workers lobbying for export restrictions. C) domestic workers lobbying for free trade. D) domestic consumers lobbying for export restrictions. E) domestic consumers lobbying for import restrictions.

A

The two industries most commonly receiving protection are A. agriculture and clothing B. agriculture and steel C. automobiles and pharmaceuticals D. pharmaceuticals and steel.

A

Under Purchasing Power Parity A) E$/E = PUS/PE. B) E$/E = PE/PES. C) E$/E = PUS + PE. D) E$/E = PUS - PE. E) E$/P = PUS/PE.

A

Under U.S. commercial policy, the escape clause results in A) temporary quotas granted to firms injured by import competition. B) tariffs that offset export subsidies granted to foreign producers. C) a refusal of the U.S. to extradite anyone who escaped political oppression. D) tax advantages extended to minority-owned exporting firms. E) tariff advantages extended to certain Caribbean countries in the U.S. market.

A

Under sticky prices A) a fall in the money supply raises the interest rate to preserve money market equilibrium. B) a fall in the money supply reduces the interest rate to preserve money market equilibrium. C) a fall in the money supply keeps the interest rate intact to preserve money market equilibrium. D) a fall in the money supply does not affect the interest rate in the short run, only in the long run. E) a fall in the money supply raises the interest rate to preserve money market equilibrium in the long run.

A

Unlike the eight previous rounds of world trade negotiations since the creation of GATT in 1948, the Doha round appears to be the first to have A. broken down with no agreement. B. received unanimous approval. C. pitted the U.S. against its traditional allies. D. forced the rich nations to compensate the poor nations.

A

For most macroeconomists A) gross national income and gross national product are the same. B) gross national income exceeds gross national product. C) gross national product exceeds gross national product. D) it is hard to tell whether gross national income equal gross national product. E) gross national product is much more important than gross national income.

A) gross national income and gross national product are the same.

Why is a current account surplus equivalent to foreign​ investment?

A current account surplus leads to the net accumulation of foreign assets

Which of the following assets is the least​ liquid?

A house

Which of the following is not a positive of having a large trade​ deficit?

A large trade deficit accumulates foreign debt that must be serviced in the future

What is​ M1?

A monetary aggregate that includes currency in circulation and checkable deposits.

A closed economy A) can save either by building up its capital stock or by acquiring foreign wealth. B) can save only by building up its capital stock. C) can save only by acquiring foreign wealth. D) cannot save either by building up its capital stock or by acquiring foreign wealth. E) can save by avoiding excessive imports.

Answer B

Every international transaction automatically enters the balance of payments A) once either as a credit or as a debit. B) twice, once as a credit and onceas a debit. C) once as a credit. D) twice, both times as debit. E) the times, once as a credit, onces as a debit, and once as an exchange.

Answer B

For open economies, A) S = I. B) S = I + CA. C) S = I -CA. D) S > I + CA. E) S < I + CA

Answer B

What is a pollution haven?

A pollution haven is a country with relatively lax environmental standards or enforcement. In effect, countries with strict standards export activities that pose high environmental risks to countries that are willing to accept the risks.

Explain why (holding interest rates constant), a rise in the expected depreciation in a country's currency leads to depreciation of that currency today.

A rise in the expected depreciation rate of the dollar raises the expected dollar return on euro deposits. Now, there are excess supply of dollar deposits (euro deposits offer higher expected rate of return than do dollar deposits). The dollar must depreciate to remove this excess supply.

Based on purchasing power parity​ (PPP), which of the​ following, all else being​ equal, could lead to a​ long-run real appreciation of the U.S.​ dollar?

A rise in the growth rate of the U.S. GDP.

Explain what a "vehicle currency" is, and why is the US dollar considered one?

A vehicle currency is one that is widely used to denominate international contracts made by parties who do not reside in the country that issues the vehicle currency. Since in 2004, nearly 90% of foreign exchange transactions involve exchanges of foreign currencies for US dollars, therefore, it is considered a vehicle currency.

Which of the following is NOT an example of debt finance in developing​ countries?

Citibank purchases shares of a Brazilian firm on the stock market

Gross National Product represents the sum of the following expenditure​ categories:

Consumption, investment, government purchases, and the current account balance

The U.S. government sells gold for dollars.

Credit in the financial account

1) The existence of positive externalities due to the impossibility of full appropriability A) supports the conclusions of the Heckscher-Ohlin model. B) rejects the usefulness of government protectionism. C) supports the concept that the government should support only high-tech industries. D) provides support for government protectionism. E) supports arguments for free trade.

D

The highest component of GNP is A) the current account. B) investment. C) government purchases. D) consumption. E) trade.

D) consumption.

A change in the level of the supply of money A) increases the long-run values of the interest rate and real output. B) decreases the long-run values of the interest rate and real output. C) has no effect on the long-run values of the interest rate, but may affect real output. D) has no effect on the long-run values of real output, but may affect the interest rate. E) has no effect on the long-run values of the interest rate and real output.

E

cont.^ As the result of a decline in relative demand for the US​ goods, the dollar will __________ in real terms. As a result of this​ change, the dollar will also __________ in nominal terms in the​ long-run.

Depreciate Depreciate (new relative demand line will shift left from the initial)

Suppose a specific factors economy produces two goods: x and y. Given that the economy is open to trade, and assuming that D in consumption, Q is production, and P is price, the budget constraint can be defined as...

Dx-Qx=(Py/Px)(Qy-Dy)

2) The United States A) does not provide more support for R&D as compared to other forms of investment. B) provides support for R&D by imposing high tariffs on R&D intensive products. C) provides support for R&D by providing direct subsidies for such activities. D) provides support for R&D through tax legislation. E) provides support for R&D through grant incentives.

D

A change in the money supply creates demand and cost pressures that lead to future increases in the price level from which main sources? I. Excess demand for output and labor II. Inflationary expectations III. Raw materials prices A) I B) II C) II and III D) I and II E) I, II, and III

E

A family's summer house on Cape Cod pays a return in the form of A) interest rate. B) capital gains. C) the pleasure of vacations at the beach. D) stock options. E) capital gains and pleasure.

E

Government transfer payments like social security and unemployment benefits are A) included in government purchases. B) not included in government purchases. C) not included in government purchases, but they are included in the consumptioncomponent of GNP. D) not included in government purchases, but they are part of the investment component of GNP. E) included in government purchases but not in the GNP.

Answer B

In 1929, government purchases accounted for A) only 18.5 percent of U.S. GNP. B) only 8.5 percent of U.S. GNP. C) 28.5 percent of U.S. GNP. D) 38.5 percent of U.S. GNP. E) 48.5 percent of U.S. GNP.

Answer B

In a closed economy, national saving A) sometimes equals investment. B) always equals investment. C) is always less than investment. D) is always more than investment. E) is never equal to investment.

Answer B

In a closed economy, private saving, , is equal to A) I -(G -T). B) I + (G -T). C) I + (G + T). D) I -(G + T).E) I + (G -T) + C

Answer B

Movements in GDP A) and GNP usually do not differ greatly. B) and GNP usually do not differ greatly, as a practical matter. C) and GNP usually do differ greatly. D) are usually smaller than those of GNP movements, in practice. E) are inversely proportional to movements in GNP

Answer B

Movements in GDP A) differ greatly from movements in GNP. B) do not differ greatly from movements in GNP. C) are not allowed to differ at all from movements in GNP by definition. D) need to be inflation adjusted in order to match movements in GNP. E) are not relevant to an examination of national income.

Answer B

Purchases of inventories by A) firms are not counted in investment spending. B) firms are also counted in investment spending. C) households are also counted in investment spending. D) households and Firms are also counted in investment spending. E) foreign consumers are counter in investment spending.

Answer B

The position of the United States current account balance in 2009 was A) lent over 6 percent of its GNP, resulting in a large current account surplus. B) borrowed over 9 percent of its GNP, leadingto a large current account deficit. C) achieved a currant account balance of zero. D) borrowed over 10 percent of its GNP, leading to a large current account deficit. E) borrowed less then 5 percent of its GNP, leading to a large current account surplus.

Answer B

Which one of the following statements is the MOST accurate? A) GNP plus depreciation is called net national product (NNP). B) GNP less depreciation is called net national product (NNP). C) GNP less depreciation is called net factor product (NFP). D) GDP plus depreciation is called net national product (NNP). E) GDP less depreciation is called net national product (NNP).

Answer B

Disposable income is National income A) less taxes collected from households and firms by the government. B) plus net taxes collected from households and firms by the government. C) less net taxes collected from firms by the government. D) less net taxes collected from households by the government. E) less net taxes collected from households and firms by the government.

Answer E

In the United States over the past fifty years, the fraction of GNP devoted to consumption has fluctuated in a range of about A) 42 to 49 percent. B) 32 to 39 percent. C) 22 to 29 percent. D) 82 to 89 percent .E) 62 to 70 percent.

Answer E

National income equals GNP A) less depreciation, less net unilateral transfers, less indirect business taxes. B) less depreciation, plus net unilateral transfers, plus indirect business taxes. C) less depreciation, less net unilateral transfers, plus indirect business taxes. D) plus depreciation, plus net unilateral transfers, less indirect business taxes. E) less depreciation, plus net unilateral transfers, less indirect business taxes.

Answer E

Net unilateral transfers A) are part of a national income. B) are part of a country's product. C) must be added to NNP in calculations of national income. D) are part of a country's GNP. E) Only A and C.

Answer E

Ricardian equivalence argues that when the government A) increases taxes and raises its deficit, consumers anticipate that they will face higher taxes later to pay for the resulting government debt, thus people will raise their own private saving to offset the fall in government saving. B) cuts taxes and decreases its deficit, consumers anticipate that they will face higher taxes later to pay for the resulting government debt, thus people will raise their own private saving to offset the fall in government saving .C) cuts taxes and raises its surplus, consumers anticipate that they will face higher taxes later to pay for the resulting government debt, thus people will raise their own private saving to offset the fall in government saving. D) cuts taxes and raises its deficit, consumers anticipate that they will face lower taxes later to pay for the resulting government debt, thus people will raise their own private saving to offset the fall in government saving. E) cuts taxes and raises its deficit, consumers anticipate that they will face higher taxes later to pay for the resulting government debt, thus people will raise their own private saving to offset the fall in government saving

Answer E

The United States began to report its gross domestic product (GDP) only since A) 1900. B) 1921. C) 1931. D) 1941. E) 1991.

Answer E

The earnings of a Spanish factory with British owners are A) counted in Spain'sGDP. B) part of Britain's GNP. C) only counted in Britain's GDP. D) only part of Spain's GNP. E) counted in Britain's GDP and are a part of Spain's GNP

Answer E

Unilateral transfers between countries are A) long-term loans .B) only international gifts, never payments that do not correspond to the purchase of any good, service, or asset. C) part of the current account but not a part of national income. D) known for reducing the income of capital owners. E) the difference between Y and GNP if the identity Y = C + I + G + CA holds exactly.

Answer E

Which of the following is FALSE about private savings and government savings? A) SP= Y -T -C B) Unlike private saving decisions, government saving decisions are often made with an eye toward their effect on output and employment. C) Total savings (S) = SP + . D) The national income identity can help us to analyze the channels through which government saving decisions influence macroeconomic conditions. E) None of the above; all statements are true.

Answer E

A reduction in a country's money supply causes A) its currency to depreciate in the foreign exchange market. B) its currency to appreciate in the foreign exchange market. C) does not affect its currency in the foreign market. D) does affect its currency in the foreign market in an ambiguous manor. E) affects other countries currency in the foreign market.

B

According to the rules of the World Trade Organization​ (WTO), countries are free to impose whatever environmental regulations they​ choose, as long as the regulations A. provide exemptions for the​ world's low-income countries B. are nondiscriminatory C. do not hinder worldwide economic growth D. are sanctioned by Greenpeace.

B

An increase in a country's money supply causes A) its currency to appreciate in the foreign exchange market while a reduction in the money supply causes its currency to depreciate. B) its currency to depreciate in the foreign exchange market while a reduction in the money supply causes its currency to appreciate. C) no effect on the values of it currency in international markets. D) its currency to depreciate in the foreign exchange market while a reduction in the money supply causes its currency to further depreciate. E) its currency to depreciate in the domestic market and appreciate in the foreign market.

B

An increase in the world relative demand for U.S. output causes A) a short-run real depreciation of the dollar against the euro. B) a long-run real appreciation of the dollar against the euro. C) a long-run real depreciation of the dollar against the euro. D) a short-run real appreciation of the euro against the dollar. E) a long-run real appreciation of the euro against the dollar.

B

An individual's need for liquidity would increase if A) the average value of transactions carried out by the individual fell. B) the average value of transactions carried out by the individual rose. C) the individual got a raise. D) the individual received a new ATM card. E) the individual wanted to avoid risks.

B

Countervailing duties are intended to neutralize any unfair advantage that foreign exporters might gain because of foreign A) tariffs. B) subsidies. C) quotas. D) Local-Content legislation. E) comparative advantage.

B

Do data on the U.S. official settlements balance give an accurate picture of the extent to which foreign central banks buy and sell dollars in currency markets? A. No, this account does not include sales or purchases of dollars by foreign central banks. B. No, this account provides only a partial picture because it shows a net value of all transactions. C. Yes, this account balance is equal to the total net sales of dollars by foreign central banks. D. Yes, this account balance is equal to the total net purchases of dollars by foreign central banks.

B

Most developing countries oppose including labor standards in trade agreements because A) they believe this would involve a loss of their national sovereignty. B) they believe this would limit their ability to export to rich markets. C) they believe this would create an uneven playing field. D) multinational corporations control them. E) they do not want to improve wages for their workers.

B

National income equals GNP A. less depreciation or net unilateral transfers. B. less depreciation, plus net unilateral transfers. C. less depreciation, less net unilateral transfers. D. plus depreciation, plus net unilateral transfers. E. plus depreciation, less net unilateral transfers.

B

Which of the following is NOT a valid argument against a strategic trade policy in an​ externality-generating sector? A. The externalities are hard to measure. B. The externalities can spill over across the border if free trade is allowed. C. Such policies can lead to trade wars. D. It might be better to address the appropriability problem directly.

B

Which of the following is not a key assumption that allows strategic trade policy to work effectively in the​ Brander-Spencer example of Airbus and​ Boeing? A. The economies of scale are large for each firm. B. A subsidy enables both firms to operate profitably. C. Firms can earn excess returns. D. Only one firm can profitably produce aircraft.

B

When one applies the Heckscher-Ohlin model of trade to the issue of trade-related income redistributions, one must conclude that North South trade, such as U.S.-Mexico trade A) must help low skill workers on both sides of the border. B) is likely to hurt high-skilled workers in the U.S. C) is likely to hurt low-skilled workers in the U.S. D) is likely to hurt low-skilled workers in Mexico. E) is likely to help highly skilled workers in Mexico.

C

Which one of the following statements is the MOST accurate? A) GNP plus depreciation is called net national product (NNP). B) GNP less depreciation is called net national product (NNP). C) GNP less depreciation is called net factor product (NFP). D) GDP plus depreciation is called net national product (NNP). E) GDP less depreciation is called net national product (NNP).

B) GNP less depreciation is called net national product (NNP).

In a closed economy, private saving, , is equal to A) I - (G - T). B) I + (G - T). C) I + (G + T). D) I - (G + T). E) I + (G - T) + C.

B) I + (G - T).

21) Which of the following statements is MOST accurate? A) In the output market, an increase in demand for U.S. output leads to an increase in the long-run nominal dollar/euro exchange rate. B) In the output market, an increase in the demand for European output leads to an increase in the long-run nominal dollar/euro exchange rate. C) In the output market, a decrease in demand for U.S. output leads to a decrease in the long-run nominal dollar/euro exchange rate. D) In the output market, an increase in the demand for European output leads to a decrease in the long-run nominal dollar/euro exchange rate. E) In the output market, an increase in the demand for European output leads to an increase in the long-run nominal euro/dollar exchange rate.

B) In the output market, an increase in the demand for European output leads to an increase in the long-run nominal dollar/euro exchange rate.

A(n) ________ of a nation's currency will cause imports to ________ and exports to ________, all other things held constant. A) depreciation; increase; decrease B) appreciation; decrease; increase C) depreciation; decrease; increase D) appreciation; increase; increase E) depreciation; decrease; decrease

C) depreciation; decrease; increase

Export embargoes cause greater losses to consumer surplus in the target country A) the lesser its initial dependence on foreign produced goods. B) the more elastic is the target country's demand schedule. C) the more elastic is the target country's domestic supply. D) the more inelastic the target country's supply. E) the larger the target country's labor force is.

D

In open economies, A. as in a closed economy, saving and investment are not necessarily equal. B. investment always refers to the domestic stock market. C. saving and investment are not necessarily equal as they are in a closed economy. D. saving and investment are necessarily equal. E. saving and investment are necessarily equal contrary to the case of a closed economy.

C

When are preferential trade agreements​ welfare-improving? A. When they take a form of free trade zone B. When they take a form of customs union C. When they lead to trade creation D. When they lead to trade diversion.

C

The opportunity to exploit economies of scale is one of the gains to be derived by removing tariffs and other trade distortions. These gains will be the result of a decrease in A) world prices of imports. B) the consumption distortion loss triangle. C) the production distortion loss triangle. D) international labor mobility. E) excessive entry and inefficient business practices.

E

The optimum tariff is A) the best tariff a country can obtain via a WTO negotiated round of compromises. B) the tariff, which maximizes the terms of trade gains. C) the tariff, which maximizes the difference between terms of trade gains and terms of trade loses. D) not practical for a small country due to the likelihood of retaliation. E) not practical for a large country due to the likelihood of retaliation.

E

The reason Airbus succeeded in the Brander Spencer example is that A) Boeing made the first move in this strategic game. B) Europeans tend to be better strategists than corn-fed Americans. C) the Airbus actually was a better plane than the Boeing 747. D) U.S. laws actually prohibit U.S. exporters from bribing foreign officials. E) the subsidy removed the advantage that Boeing gained with their head start in production.

E

Under a flexible-price monetary approach to the exchange rate A) when the domestic money supply falls, the price level would eventually fall, increasing the interest rate. B) when the domestic money supply falls, the price level would fall right away, causing a reduction in the interest rate. C) when the domestic money supply falls, the price level would fall right away, causing an increase in the interest rate. D) when the domestic money supply falls, the price level would eventually fall, keeping the interest rate constant. E) when the domestic money supply falls, the price level would fall right away, keeping the interest rate constant.

E

In the United States over the past fifty years, the fraction of GNP devoted to consumption has fluctuated in a range of about A) 42 to 49 percent. B) 32 to 39 percent. C) 22 to 29 percent. D) 82 to 89 percent. E) 62 to 70 percent.

E) 62 to 70 percent.

Ricardian equivalence argues that when the government A) increases taxes and raises its deficit, consumers anticipate that they will face higher taxes later to pay for the resulting government debt, thus people will raise their own private saving to offset the fall in government saving. B) cuts taxes and decreases its deficit, consumers anticipate that they will face higher taxes later to pay for the resulting government debt, thus people will raise their own private saving to offset the fall in government saving. C) cuts taxes and raises its surplus, consumers anticipate that they will face higher taxes later to pay for the resulting government debt, thus people will raise their own private saving to offset the fall in government saving. D) cuts taxes and raises its deficit, consumers anticipate that they will face lower taxes later to pay for the resulting government debt, thus people will raise their own private saving to offset the fall in government saving. E) cuts taxes and raises its deficit, consumers anticipate that they will face higher taxes later to pay for the resulting government debt, thus people will raise their own private saving to offset the fall in government saving.

E) cuts taxes and raises its deficit, consumers anticipate that they will face higher taxes later to pay for the resulting government debt, thus people will raise their own private saving to offset the fall in government saving.

16) Interest rate differences between countries depend on A) differences in expected inflation, but not on expected changes in the real exchange rate. B) differences in expected changes in the real exchange rate, but not on expected inflation. C) neither differences in expected inflation, nor on expected changes in the real exchange rate. D) differences in expected inflation and nothing else. E) differences in expected inflation, and on expected changes in the real exchange rate.

E) differences in expected inflation, and on expected changes in the real exchange rate.

When a country's currency depreciates A) foreigners find that its exports are more expensive, and domestic residents find that imports from abroad are more expensive. B) foreigners find that its exports are more expensive, and domestic residents find that imports from abroad are cheaper. C) foreigners find that its exports are cheaper; however, domestic residents are not affected. D) foreigners are not affected, but domestic residents find that imports from abroad are more expensive. E) foreigners find that its exports are cheaper and domestic residents find that imports from abroad are more expensive.

E) foreigners find that its exports are cheaper and domestic residents find that imports from abroad are more expensive.

11) A change in the level of the supply of money A) increases the long-run values of the interest rate and real output. B) decreases the long-run values of the interest rate and real output. C) has no effect on the long-run values of the interest rate, but may affect real output. D) has no effect on the long-run values of real output, but may affect the interest rate. E) has no effect on the long-run values of the interest rate and real output.

E) has no effect on the long-run values of the interest rate and real output.

In 1948, _ formed as an international treaty with objective to remove trade _ and increase world _ through _ reductions _ _ _ (MFN) clause: -principle of non-_ of all trading partners by a _, by granting the _ rate the country imposes to its _ _ country to all other countries

GATT; barriers; trade; tariff; most favored nation; discrimination; country; tariff; most favored;

Labels for exports

Has been partially successful in telling consumers that a good is produced in humane and environmentally friendly ways.​ However, involves some intrusion by foreign inspectors that is perceived as an infringement of sovereignty.

Suppose that output demand and supply trends induce people to expect​ Home's currency to depreciate in real terms against the currency of​ Foreign, its key trading partner. This implies that the expected real interest rate should be

Higher in Home (bc if IR increases => Exchange rates increase, depreciating the $)

Discuss the values of private saving in closed and open economies.

In a closed economy, private saving, , is equals to, I + (G -T). In a open economy, private saving, , is equals to I + CA + (G -T). Open economy helps in extending the opportunities for private saving or dis-saving, or borrowing

How do we distinguish in the model between the short run and the long​ run?

In the short run price level is​ fixed; in the long​ run, it is flexible.

In 1994, _ _ agreement ratified by major nations -further _ cuts -_ dispute _ mechanism -phasing out of _ _ by developed countries -phasing in of _ _ (IPR) protection by _ countries -phasing out of _ export _, quotas, and tariffs by _ countries -introduction of rules over _ trade -formation of the _ In 2001, _ round negotiations but came to standstill in 2008

Uruguay Round; tariff; dumping; resolution; textile quotas; intellectual property; developing; agricultural; subsidies; developed; services; WTO; Doha;

The monetary approach to the exchange rate predicts that the dollar will depreciate in the long run​ if, ceteris paribus​,

US interest rate rises or European interest rate falls

Big Mac Index

Using the price of a big mac in different countries to be able to identify the difference in exchange rates. If the prices are not the same we know whether currencies reflect PPP or not. This method essentially uses a big mac as the "basket of goods" to compare

In an open economy holding GNP and consumption spending constant and where private savings equals domestic​ investment, a government budget deficit must be matched by

a current account deficit.

After September​ 11, 2001, the U.S. government imposed greater restrictions on Student ​(​F-1, ​J-1​) visas making it more difficult for international students to enter the U.S. for undergraduate and graduate study. This is likely to cause ??? in real tuition rates​ (the price of​ education) among trading nations.

a divergence

What is the essence of the​ "terms-of-trade" argument against free​ trade?

a large country can improve its terms of trade by imposing tariffs, and the optimal tariff is positive

Faced with the evidence of poor working conditions and low wages in the border maquiladoras, economists

argue that the poor conditions and low wages are actually improvements for the Mexican workers, and may be cited as gains-from-trade.

Labor standards in trade are typically opposed by most developing countries who believe that they will be used

as a protectionist tool by import-competing producers in industrial countries.

The shipbreakers of Alang are

competing with pollution-producing industries in countries outside of India.

GATT rules of _ of trade policy (pre-_ round agreement): -any new _ must be matched by an equivalent _ in existing tariffs -_ _ banned except for _ goods -_ banned except in cases of "_ _"

conduct; Uruguay; tariffs; decrease; export subsidy; agricultural; quota; market disruption

most of the world's population and a majority of countries are:

either low-income or middle income

According to WTO, country are free to impose whatever _ regulations they choose, as long as regulations are _

environment; nondiscriminatory

In theory, the equality: current account + capital account + financial account = 0 must hold. In reality a statistical discrepancy is often included to achieve this balance. Which account is the likely culprit of this discrepancy? The ____________

financial account

Working conditions for clothing workers in Bangladesh are very poor. If countries refuse to buy clothing from Bangladesh in order to encourage change, the effect is likely to be that

firms will try to comply and workers will be worse off.

When a country's currency depreciates

foreigners find that its exports are cheaper and domestic residents find that imports from abroad are more expensive

Futures contracts differ from forward contracts in that

futures contracts allow you to sell your contract on an organized futures exchange

The _ curve is the relationship showing _ to environment _ as country's per capita _ increases from initially low level but then falls with further _ in per capita income -country's _ that poses the greatest _ to planet -least afforded to pay environmental/activism _

kuzner's; damage; increases; income; increase

Export labels provide an alternative to the use of trade barriers to promote environmental standards in partner trading​ countries, since

labels might allow customers to determine whether products were produced in an environmentally sustainable fashion.

What is the main problem with imposing labor standards to prevent child labor and poor working conditions in the exporting sector of a​ less-developed country?

less-developed countries are resistant to them

Suppose the U.S. foreign assets are 69 percent of the U.S.​ GDP, and the U.S. foreign liabilities are 96 percent of the U.S. GDP. ​ Moreover, suppose that 68 percent of U.S. foreign assets are denominated in foreign​ currencies, while all liabilities to foreigners are denominated in U.S. dollars. How will a 11 percent depreciation of the dollar affect ​foreigners' net foreign claims on the U.S. measured in U.S. dollars​ (as a percent of U.S.​ GDP)? ​(You will need a calculator. Round your answer to the whole percentage​ point, no decimals​.) Foreigners will experience a net capital ___ equal to ___ percent of U.S. GDP.

loss.....5%

If the goods' money prices do not change, an appreciation of the dollar against the pound

makes British sweaters cheaper in terms of American jeans

If the goods' money prices do not change, an depreciation of the dollar against the pound

makes British sweaters more expensive in terms of American jeans

A depreciation of a country's currency makes its goods

makes its goods cheaper for foreigners

In today's world markets, poor developing countries tend to rely primarily on exports of

manufactured products.

By April 2010

only about 39 percent of foreign exchange trades were against euros

Which of the following is not one of the three ways for countries to handle different standards​ abroad?

optimization standards

If there is an excess supply of money:

the interest rate falls.

The exchange rate between currencies depends on

the interest rate that can be earned on deposits of those currencies and the expected future exchange rate.

The aggregate money demand depends on

the interest rate, price level, and real national income.

The WTO seems at times to be interfering in domestic policy since

the line between domestic policies and de factor protectionism is often fuzzy.

In each sector of a specific factors economy, profit-maximizing employers will demand labor up to the point where ...

the marginal product of labor times the price of the product equals the wage rate.

Criticisms of the Brander-Spencer model include all EXCEPT which of the following?

the problem of harm to interests of consumers

The action of arbitrage is

the process of buying a currency cheap and selling it dear

The International Labor Organization​ (ILO) proposes that five labor standards are basic rights. These include

the right to organize and bargain collectively, prohibition of forced labor

The future date on which the currencies are actually exchanged is called what?

the value date

For the private businesses that were interested in producing for the U.S.​ market, a​ "pro" was the chance to have

their product adopted as the standard

Most developing countries oppose including labor standards in trade agreements because

they believe this would limit their ability to export to rich markets.

Since dollar and yen interest rates are not measured in comparable terms

they can move quite differently over time

After a permanent increase in the money supply,

the exchange rate overshoots in the short run.

Individuals base their demand for an asset on

the expected return, how risky that expected return is, and the asset's liquidity.

We have the following data for a hypothetical open​ economy: GNP​ = ​$9,000 Consumption​ (C) = ​$8,000 Investment​ (I) = ​$1,000 Government Purchases​ (G) = ​$1,200 Tax Collections​ (T) = ​$1,200 What is the value of total savings​ S? What is the value of the current account balance​ CA?

$-200 $-1200

How many British pounds would it cost to buy a pair of American designer jeans costing $45 if the exchange rate is 1.80 dollars per British pound?

25 British pounds

The most extreme inflationary conditions occurred A) in Zimbabwe in 2008. B) in Chile in 2012. C) in Eastern Europe in the 1990s. D) in Western Europe in the 1980s. E) in Germany in 20013.

A

Given the following​ data: Et ​ = yen 115 ​= $1.00 E​t+1 ​ = yen 80 ​= $1.00 ​{one year later​} iJapan ​ = 15 ​% annually iU.S. ​ = 12 ​% annually Calculate the future value of a​ $1,000 investment. If the​ $1000 is invested in the​ U.S., the future value is ​??? . ​(Round your response to two decimal​ places.) If the​ $1000 is invested in Japan​ (and repatriated back to​ dollars), the future value is ​??? . ​(Round your response to two decimal​ places.)

$1120.00 $1653.13

A college textbook is selling for​ (US) $140 in the United States. That same textbook sells in Canada for​ (CA) $150. The exchange rate is​ (CA) $1.10​ = (US)​ $1.00. Shipping costs are​ (US) $5.00. Ignoring the shipping costs. What is the U.S. price of the textbook purchased in​ Canada? (US) ​??? ​ (Enter your response rounded to the nearest penny.​) What is the Canadian price of the textbook purchased in the​ U.S.? (CA) ​??? ​(Enter your response rounded to the nearest penny.​) When you take shipping costs into​ account, are textbooks likely to be purchased in the U.S. and sold in​ Canada? A. Yes B. No When you take shipping costs into​ account, are textbooks likely to be purchased in Canada and sold in the​ U.S.? A. Yes B. No

$136.36 $154.00 NO NO

Given the following Balance of Payment data for a given​ country: Current Account​ Balance: ​$−3,600 Capital Account​ Balance: ​$−125. What must be the Financial Account​ Balance:

$3725 The Balance of​ Payment, by​ definition: current account​ + financial account​ + capital account​ = 0. 0=(-3600)+FA+(-125)

We have the following data for a hypothetical open​ economy: GNP​ = ​$14,000 Consumption​ (C) = ​$7800 Investment​ (I) = ​$1,200 Government Purchases​ (G) = ​$1,200 What is the value of the current account​ balance?

$3800 Y​ = C​ + I​ + G​ + EX-​IM, and CA​ = EX−​IM 12000=7500+800+1600+CA

How many dollars would it cost to buy an Edinburgh Woolen Mill sweater cost in 50 British pounds if the exchange rate is 1.25 dollars per one British pound?

$62.50

How many dollars would it cost to buy an Ediburgh Woolen Mill sweater cost in 50 British pounds if the exchange rate is 1.50 dollars per one British pound?

$75.00

How many dollars would it cost to buy an Edinburgh Woolen Mill sweater cost in 50 British pounds if the exchange rate is 1.80 dollars per one British pound?

$90.00

What influences Real Exchange Rate?

*A change in relative demand: an increase in demand drives up the price value which changes the real exchange equation *Same idea as above but a change in relative supply, an increase would cause the price to drop thus also changing the equation

Predictions of change:

*Increase in money supply => depreciation of $ *Increase in interest rates => depreciation of $ *Increase of outputs/production => appreciates $

Predictions about changes

*Money supply: a permanent rise in the domestic money supply causes a proportional increase in the domestic price level, thus causing a proportional depreciation in the domestic currency (through PPP). This is same prediction as long-run model without PPP. *Interest rates: a rise in domestic interest rates lowers the demand of real monetary assets, and is associated with a rise in domestic prices, thus causing a proportional depreciation of the domestic currency (through PPP).

Cont.

*Output level: a rise in the domestic level of production and income (output) raises domestic demand of real monetary assets, and is associated with a decreasing level of average domestic prices (for a fixed quantity of money supplied), thus causing a proportional appreciation of the domestic currency (through PPP).

Exchange rates:

*The "floating" or Nominal exchange rate does not always reflect PPP and changes in price (inflation) *Real exchange rates adjust to reflect PPP and price changes

E rising/falling (E=exchange rate)

*if E rises that is considered a depreciation *if E falls it is considered an appreciation

Why does PPP fail or not hold true?

*in the short run: 1.Trade barriers and nontradable products 2. Imperfect competition 3. Differences in measures of average prices for baskets of goods and services

Purchasing Power Parity (PPP)

*is the application of the law of one price across countries for all goods and services, or for representative groups ("baskets") of goods and services. *implies that the exchange rate is determined by levels of average prices

Real Exchange Rate approach

*q is the real exchange rate *it is = to the exchange rate times the Price level in the foreign country divided by the Price level in the US

Monetary approach to the exchange rate

*uses monetary factors to predict how exchange rates adjust in the long run, based on the absolute version of PPP. *It predicts that levels of average prices across countries adjust so that the quantity of real monetary assets supplied will equal the quantity of real monetary assets demanded:

Who are the major participants in the foreign exchange market?

- commercial banks - corporations - nonbank financial institutions - central banks

What are the three factors that affect the demand for foreign currency?

- expected return - risk - liquidity

What are examples of financial derivatives?

- forwards - swaps - futures - options

Statistical discrepancy is

-1 x (Current account + Financial account + capital account)

What is the expected dollar rate of return on euro deposits if today's exchange rate is $1.167 per euro, next year's expected exchange rate is $1.10 per euro, the dollar interest rate is 10% and the euro interest rate is 5%?

-1%

What are ways to realize the benefits of trade and resolve standards​ conflicts?

-Require labels for exports. -Used home country standards for home country firms operating in a foreign country. -Use international organizations as a forum for negotiations.

Financial account balance equals

-current account balance -Capital account balance 1400+125=-1275

Which of the following is not part of the definition for Gross National​ Product?

..produced within a​ country's borders...

What is the exchange rate between the dollar and the British pound if a pair of American jeans costs 50 dollars in New York and 100 Pounds in London?

0.5 dollars per British pound

Using Big Mac Index

1.Convert currencies (15,000Rb=$1, then $1,000=15,000,000Rb) 2.Purchase big macs using above # divided by the price of big macs given in a table 3.Sell them in foreign market using the number found in part 2 times the $ rate given in table

Calculating the future value of an investment if it is invested in a foreign country:

1.Multiply the initial investment by the Et rate (spot) 2.Then multiply that amount by (1+the foreign interest rate) 3.Last divide that amount by the Et+1 rate (future)

At the beginning of 2012, you pay $100 for a share of stock that then pays you a dividend of $1 at the beginning of 2013. If the stock price rises from $100 to $109 per share over the year, then you have earned an annual rate of return of

10 percent

What is the expected dollar rate of return on euro deposits if today's exchange rate is $1.10 per euro, next year's expected exchange rate is $1.16 per euro, the dollar interest rate is 10% and the euro interest rate is 5%?

10%

What is the expected dollar rate of return on euro deposits if today's exchange rate is $1.10 per euro, next year's expected exchange rate is $1.166 per euro, the dollar interest rate is 10% and the euro interest rate is 5%?

11%

Given the following​ data: Upper E Subscript t ​= yen 115 ​= $1.00 Upper E Subscript t plus 1 Superscript e Baseline nbsp ​= yen 120 ​= $1.00 R U.S.​ = 8 ​% Assuming that Japan is the domestic​ currency, if the interest parity condition is expected to​ hold, interest rates in Japan ​(RJapan​) should equal ?? ​% ​ (Enter your answer as a percentage rounded to two decimal places​).

12.35

We have the following data for a hypothetical open​ economy: GNP​ = ​$12,000 Consumption​ (C) = ​$8,200 Investment​ (I) = ​$800 Government Purchases​ (G) = ​$1,600 y= C+I+G+EX-IM

1400

The Economist magazine is famous for its publication of the Big Mac indexlong dash a table of Big MacTM prices in different countries around the world. The use of the Big Mac allows for a highly standardized product sold throughout the world. Given the following abbreviated​ table: Country PriceBig Mac China Rb​ 10,000 U.K. pound 1.25 U.S. ​$2.50 Suppose that the exchange rate between China and the U.K​ is: Rb​ (Rembini) 15,000​ = pound 1.00 and that the Big MacTM could be used as a standardized commoditylong dash easily transported and not perishable. Complete the​ following: ​1,000 ​= Rb ​ = Big MacsTM ​= pound

15000000 1500 1875

What is the exchange rate between the dollar and the British pound if a pair of American jeans costs 60 dollars in New York and 30 Pounds in London?

2 dollars per British pound

How many British pounds would it cost to buy a pair of American designer jeans costing $45 if the exchange rate is 2.00 dollars per British pound?

22.5 British pounds

Government savings, , is equal to A) T - G. B) T + G. C) T = G. D) T + G - I. E) T - G = I.

A) T - G.

How many British pounds would it cost to buy a pair of American designer jeans costing $45 if the exchange rate is 1.60 dollars per British pound?

28.125 British pounds

How many British pounds would it cost to buy a pair of American designer jeans costing $45 if the exchange rate is 1.50 dollars per British pound?

30 British pounds

Consider the following hypothetical data for an open economy​ (in millions): Assets owned inside the U.S. by U.S. citizens​ = ​$140,000 Assets owned outside the U.S. by U.S. citizens​ = ​$22,786 Assets owned outside the U.S. by foreign citizens​ = ​$110,000 Assets owned inside the U.S. by foreign citizens​ = ​$19,888 The value of the International Investment Position​ (IIP) of the U.S. is

3042

In 2010, about

85 percent of foreign exchange transactions involved exchanges of foreign currencies for US dollars.

Given the following​ data: Upper E Subscript t ​= yen 115 ​= $1.00 Upper E Subscript t plus 1 Superscript e Baseline nbsp ​= yen 119 ​= $1.00 R U.S.​ = 6 ​% Assuming that Japan is the domestic​ currency, if the interest parity condition is expected to​ hold, interest rates in Japan ​(RJapan​) should equal ??? ​% ​ (Enter your answer as a percentage rounded to two decimal places​).

9.48

In an open economy, the CA is equal to A. Y - (C -I + G). B. Y - (C + I + G). C. Y + (C - I - G). D. Y + (C + I + G). E. Y - (C + I - G).

???

After a permanent increase in the money supply A) the exchange rate overshoots in the short run. B) the exchange rate overshoots in the long run. C) the exchange rate smoothly depreciates in the short run. D) the exchange rate smoothly appreciates in the short run. E) the exchange rate remains the same.

A

Customs unions differ from free trade areas by the manner in which A. member countries apply tariffs to nonmembers. B. members share the gains from trade with each other. C. countries attain membership. D. the WTO adjudicates disputes between the members.

A

During hyperinflation, exploding inflation causes real money demand to A) fall over time, and this additional monetary change makes money prices rise even more quickly than the money supply itself rises. B) increase over time, and this additional monetary change makes money prices rise even more quickly than the money supply itself rises. C) fall over time, and this additional monetary change makes money prices decrease even more quickly than the money supply itself rises. D) increase over time, and this additional monetary change makes money prices decrease even more quickly than the money supply itself rises. E) fall over time, and this additional monetary change makes money prices decrease even less quickly than the money supply itself rises.

A

Free trade and globalization is generally believed A) to cause a degradation in the world's environment. B) to improve the environment by correcting for distortions caused by import competing policies. C) to help spread the best of each country's culture, so as to uplift global cultural standards. D) to help each country safeguard the best of its own culture. E) to make no difference in the economic welfare of the world.

A

GNP accounts avoid double counting by including only the value of final goods and services sold on the market. Should the measure of imports and exports used in the GNP accounts therefore be defined to include only imports and exports of final goods and services received from and sold to other countries? A. No, total values and imports and exports should be included in the calculation of the GNP. B. Yes, only the value of final goods that are exported or imported should be included. C. Total value of imports should be included, but only the value of exports of final goods. D. Total value of exports should be included, but only the value of imports of final goods.

A

If people expect relative PPP to hold A) the difference between the interest rates offered by dollar and euro deposits will equal the difference between the inflation rates expected, in the United States and Europe, respectively, over the relevant horizon. B) the difference between the interest rates offered by dollar and euro deposits will equal the difference between the inflation rates expected in Europe and the United States, respectively. C) the difference between the interest rates offered by dollar and euro deposits will equal the difference between the inflation rates expected, over the relevant horizon, in the United States and Europe, respectively, in the short run. D) the difference between the interest rates offered by dollar and euro deposits will be above the difference between the inflation rates expected, over the relevant horizon, in the United States and Europe, respectively. E) the difference between the interest rates offered by dollar and euro deposits will be below the difference between the inflation rates expected, over the relevant horizon, in the United States and Europe, respectively.

A

If there is an excess supply of money A) the interest rate falls. B) the interest rate rises. C) the real money supply shifts left to make an equilibrium. D) the real money supply shifts right to make an equilibrium. E) the interest rate stays constant, but consumer confidence falters.

A

In 2015, the United States had A. a deficit in the current account. B. a balance in the current account. C. a positive balance of net financial flows. D. a surplus in the current account. E. From 2015 data, it is too difficult to determine whether a surplus or a deficit existed in the current account.

A

In a classic paper, Columbia University economist Phillip Cagan drew the line between inflation and hyperinflation at an inflation rate of A) 50 percent per month. B) 10 percent per month. C) 20 percent per month. D) 5 percent per month. E) 25 percent per month.

A

In early​ 2016, negotiators from 12 countries around the Pacific​ Rim, including the United States and​ Canada, agreed on a proposal for a new trade agreement called the​ Trans-Pacific Partnership​ (TPP). However, the U.S. withdrew from that agreement on January​ 23, 2017. The U.S. decision to withdraw was based A. in part on uncertainty over the likely benefits of the TPP for the United​ States, and in part on a widespread backlash against globalization that was visible in 2016 B. on provisions of the TPP that would weaken intellectual property rights and make it more difficult to resolve arguments between private businesses and national governments C. ​​on the failure of the TPP to reduce tariffs or eliminate import quotas as much as previous trade agreements had done D. on the fear that the TPP would harm the interests of both corporations and workers.

A

In practice A) changes in national price levels often tell us relatively little about exchange rate movements. B) changes in national price levels raise the exchange rate. C) changes in national price levels lower the exchange rate. D) changes in national price levels often tell us about exchange rate movements. E) changes in national price levels match identical changes in the exchange rate.

A

In the United States at the end of 2012, the total money supply, M1, amounted to approximately A) 16 percent of that year's GNP. B) 20 percent of that year's GNP. C) 30 percent of that year's GNP. D) 40 percent of that year's GNP. E) 50 percent of that year's GNP.

A

It is argued that special interest groups are likely to take over and promote protectionist policies, which may lead to a decrease in national economic welfare. This argument leads to A) a presumption that in practice a free trade policy is likely to be better than alternatives. B) a presumption that trade policy should be shifted to Non-Governmental Organizations, so as to limit taxpayer burden. C) a presumption that free trade is generally a second-best policy, to be avoided if feasible alternatives are available. D) a presumption that free trade is the likely equilibrium solution if the government allows special interest groups to dictate its trade policy. E) a presumption that protectionist policies will better serve a country as a whole than free trade policies.

A

Japan's protection of its semiconductor (RAM) producers is today seen as an object lesson in A) how strategic planning may backfire and cause a large waste of resources. B) how externalities may be successfully exploited by protectionist policies. C) how excess returns may be successfully exploited by protectionist policies. D) how government intervention may create a meaningful comparative advantage. E) how monopolies can outlast government intervention.

A

Judging by the ongoing changes in tariff rates in major trading countries, the world has been experiencing a great A) trade liberalization. B) surge of protectionism. C) lack of progress in the trade-policy area. D) move towards regional integration. E) shift from export subsidies to specific tariffs.

A

Money demand behavior may A) change as a result of demographic trends or financial innovations such as electronic cash-transfer facilities. B) change only as a result of demographic trends. C) change only as a result of financial innovations such as electronic cash-transfer facilities. D) not change as a result of demographic trends or financial innovations such as electronic cash-transfer facilities. E) change as a result of demographic trends but not as a result of financial innovations such as electronic cash-transfer facilities.

A

One of the major issues that arose during the Doha round of negotiations involved complaints by ________ about ________. A) developing countries; agricultural subsidies. B) manufacturers; intellectual property C) industrialized countries; enforcement of contracts D) Eastern European countries; European Union tariffs E) South and Central American countries; domestic content requirements

A

Our usual models of trade assume that jobs lost in one industry will be offset by jobs gained in other industries. The Autor et al. paper​ argued, however, that communities that lose manufacturing jobs to imports end up losing other jobs as well. Is this a​ contradiction? The Autor et al. paper A. does not contradict our usual models because the gains and losses from trade may B. contradicts our usual models because workers are predicted to acquire different skills to benefit from trade. C. contradicts our usual models because consumer demand is predicted to increase with lower prices. D. contradicts our usual models because workers are predicted to move geographically to benefit from trade. E. does not contradict our usual models because a​ country's industries are rarely so geographically concentrated.

A

Protectionism tends to be concentrated in two sectors A) agriculture and clothing. B) high-tech and national security sensitive industries. C) capital and skill intensive industries. D) industries concentrated in the South and in the Midwest of the country. E) financial services and manufacturing based in the Midwest

A

The German government carries out an official foreign exchange intervention in which it uses dollars held in an American bank to buy French currency from its citizens. How is this accounted for in the balance of payments? A. financial account, French asset export B. current account, German good import C. current account, French good export D. financial account, German asset import E. financial account, German asset export

A

The Heckscher-Ohlin, factor-proportions model lends support to the argument that A) trade tends to worsen the conditions of unskilled labor in rich countries. B) trade tends to worsen the conditions of owners of capital in rich countries. C) trade tends to worsen the conditions of workers in poor countries. D) trade tends to worsen the conditions of workers in rich countries. E) trade tends to worsen the conditions of highly skilled workers in rich countries.

A

The Shipbreakers of Alang utilize much labor and little capital, thereby supporting the applicability of the A) factor proportions explanation of the sources of comparative advantage. B) specific factor theory of comparative advantage. C) monopolistic competition theory of comparative advantage. D) scale economies theory of comparative advantage. E) basis of the non-dumping legislation.

A

The WTO seems at times to be interfering in domestic policy since A) the line between domestic policies and de factor protectionism is often fuzzy. B) it is a supra-national organization with the power to overturn governments. C) it determines which nations may trade what with whom. D) it punishes naughty nations. E) it exempts the U.S. and other powerful member nations from many of its edicts.

A

The aggregate demand for money can be expressed by A) Md = P × L(R,Y). B) Md = L × P(R,Y). C) Md = P × Y(R, L). D) Md = R × L(P,Y). E) Md = R × L(R, P).

A

The average tariff rate dutiable imports in the United States is approximately A) less than 10 % of the value of imports. B) 15% of the value of imports. C) 20 % of the value of imports. D) 25% of the value of imports. E) more than 30% of the value of imports.

A

The effect of an export tariff on a large country is to ________ the terms of trade. A) always improve B) sometimes improve C) leave unchanged D) sometimes worsen E) always worsen

A

The existence of positive externalities due to the impossibility of full appropriability A. provides support for government protectionism. B. supports arguments for free trade. C. supports the concept that the government should support only highminus−tech industries. D. supports the conclusions of the Heckscherminus−Ohlin model. E. rejects the usefulness of government protectionism.

A

The monetary approach to interest rates assumes that the prices of goods are ________, which implies that a country's currency will ________, when nominal interest rates ________ because of ________ expected future inflation. A) perfectly flexible; depreciate; increase; higher B) perfectly flexible; appreciate; increase; higher C) immutable; depreciate; increase; higher D) immutable; appreciate; decrease; higher E) absolutely inflexible; depreciate; decrease; higher

A

Using year-by-year data from 1987-2007 shows that A) there is a strong positive relation between average Latin American money-supply growth and inflation. B) there is a strong negative relation between average Latin American money-supply growth and inflation. C) there is a strong positive relation between average Latin American money-supply growth and deflation. D) it is difficult to find a strong positive relation between average Latin American money-supply growth and inflation. E) there is a weak positive relation between average Latin American money-supply growth and inflation.

A

What particular market failure does the​ "market failure​ argument" against free trade refer​ to? A. Any market failure that occurs in the tradable sector B. Knowledge and technology spill overs C. Environmental externalities D.​​ Unemployment.

A

When the nominal dollar interest rate ________, money demand will ________, and the general price level will ________. A) increases; decrease; increase B) increases; increase; increase C) increases; decrease; decrease D) increases; increase; decrease E) decreases; increase; increase

A

Which of the following statements is MOST accurate? A) A relative expansion of U.S. output causes a long-run depreciation of the dollar against the euro, while a relative expansion of European output causes a long-run real appreciation of the dollar against the euro. B) A relative decline of U.S. output causes a long-run depreciation of the dollar against the euro, while a relative expansion of European output causes a long-run real appreciation of the dollar against the euro. C) A relative expansion of U.S. output causes a long-run appreciation of the dollar against the euro, while a relative expansion of European output causes a long-run real depreciation of the dollar against the euro. D) A relative expansion of U.S. output causes a long-run depreciation of the dollar against the euro, while a relative decline of European output causes a long-run real appreciation of the dollar against the euro. E) A relative decline of U.S. output causes a long-run depreciation of the dollar against the euro, while a relative decline of European output causes a long-run real appreciation of the dollar against the euro.

A

Which of the following statements is MOST accurate? A) In the money market, an increase in U.S. money supply level leads to a proportional increase in the long-run nominal dollar/euro exchange rate. B) In the money market, an increase in European money supply level leads to a proportional increase in the long-run nominal dollar/euro exchange rate. C) In the money market, an increase in U.S. money supply growth rate leads to a decrease in the long-run nominal dollar/euro exchange rate. D) In the money market, an increase in European money supply growth leads to an increase in the long-run nominal dollar/euro exchange rate. E) In the money market, an increase in U.S. money supply level leads to a proportional decrease in the long-run nominal dollar/euro exchange rate.

A

Which of the following statements is the MOST accurate? In general A) the monetary approach to the exchange rate is a long run theory. B) the monetary approach to the exchange rate is a short run theory. C) the monetary approach to the exchange rate is both a short and long run theory. D) the monetary approach to the exchange rate neither long run nor short run theory. E) the monetary approach to the exchange rate is considered less practical than the law of one price.

A

Which of the following statements is the MOST accurate? The law of one price states A) in competitive markets free of transportation costs and official barriers to trade, identical goods sold in different countries must sell for the same price when their prices are expressed in terms of the same currency. B) in competitive markets free of transportation costs and official barrier to trade, identical goods sold in the same country must sell for the same price when their prices are expressed in terms of the same currency. C) in competitive markets free of transportation costs and official barrier to trade, identical goods sold in different countries must sell for the same price. D) identical goods sold in different countries must sell for the same price when their prices are expressed in terms of the same currency. E) in competitive markets free of official barrier to trade, identical goods are sold at the same price regardless of transportation costs.

A

Which one of the following statements is the MOST accurate? A) A permanent increase in a country's money supply causes a proportional long-run depreciation of its currency against foreign currencies. B) A temporary increase in a country's money supply causes a proportional long-run depreciation of its currency against foreign currencies. C) A permanent increase in a country's money supply causes a proportional long-run appreciation of its currency against foreign currencies. D) A permanent increase in a country's money supply causes a proportional short-run depreciation of its currency against foreign currencies. E) A permanent increase in a country's money supply causes a proportional short-run appreciation of its currency against foreign currencies.

A

Why are international negotiations important in order to reduce tariff rates​ worldwide? A. They help avoid trade wars. B. ​Special-interest groups cannot affect international negotiations. C. Large countries can impose policy restrictions on smaller countries. D. The world as a whole gains from free trade.

A

Government purchases currently take up about A) 20 percent of U.S. GNP, and this share has not changed much since the late 1950s. B) 38 percent of U.S. GNP, and this share has not changed much since the late 1950s. C) 18 percent of U.S. GNP, and this share has been increasing since the late 1950s. D) 18 percent of U.S. GNP, and this share has been decreasing since the late 1950s. E) 25 percent of U.S. GNP, and this share has been decreasing since the late 1950s.

A) 20 percent of U.S. GNP, and this share has not changed much since the late 1950s.

Which one of the following statements is the MOST accurate? A) A depreciation of a country's currency makes its goods cheaper for foreigners. B) A depreciation of a country's currency makes its goods more expensive for foreigners. C) A depreciation of a country's currency makes its goods cheaper for its own residents. D) A depreciation of a country's currency makes its goods cheaper. E) An appreciation of a country's currency makes its goods more expensive.

A) A depreciation of a country's currency makes its goods cheaper for foreigners.

10) Which one of the following statements is the MOST accurate? A) A permanent increase in a country's money supply causes a proportional long-run depreciation of its currency against foreign currencies. B) A temporary increase in a country's money supply causes a proportional long-run depreciation of its currency against foreign currencies. C) A permanent increase in a country's money supply causes a proportional long-run appreciation of its currency against foreign currencies. D) A permanent increase in a country's money supply causes a proportional short-run depreciation of its currency against foreign currencies. E) A permanent increase in a country's money supply causes a proportional short-run appreciation of its currency against foreign currencies.

A) A permanent increase in a country's money supply causes a proportional long-run depreciation of its currency against foreign currencies.

Which one of the following expressions is the MOST accurate? A) CA = EX - IM B) CA = IM - EX C) CA = EX = IM D) CA = EX + IM E) CA - IM = EX

A) CA = EX - IM

The CA is equal to A) Y - (C - I + G). B) Y + (C + I + G). C) Y - (C + I + G). D) Y - (C + I - G). E) Y + (C - I - G).

A) Y - (C - I + G).

26) Under sticky prices A) a fall in the money supply raises the interest rate to preserve money market equilibrium. B) a fall in the money supply reduces the interest rate to preserve money market equilibrium. C) a fall in the money supply keeps the interest rate intact to preserve money market equilibrium. D) a fall in the money supply does not affect the interest rate in the short run, only in the long run. E) a fall in the money supply raises the interest rate to preserve money market equilibrium in the long run.

A) a fall in the money supply raises the interest rate to preserve money market equilibrium.

The dollar rate of return on euro deposits is A) approximately the euro interest rate plus the rate of depreciation of the dollar against the euro. B) approximately the euro interest rate minus the rate of depreciation of the dollar against the euro. C) the euro interest rate minus the rate of inflation against the euro. D) the rate of appreciation of the dollar against the euro. E) the euro interest rate plus the rate of inflation against the euro.

A) approximately the euro interest rate plus the rate of depreciation of the dollar against the euro.

A country's current account A) balance equals the change in its net foreign wealth. B) balance equals the change in its foreign wealth. C) surplus equals the change in its foreign wealth. D) deficit equals the change in its foreign wealth. E) balance equals its GNP.

A) balance equals the change in its net foreign wealth.

Which of the following is a correct prediction based on the PPP model of the exchange​ rates?

An increase in the U.S. interest rates leads to depreciation of the dollar.

30) Which of the following statements is the MOST accurate? The law of one price states A) in competitive markets free of transportation costs and official barriers to trade, identical goods sold in different countries must sell for the same price when their prices are expressed in terms of the same currency. B) in competitive markets free of transportation costs and official barrier to trade, identical goods sold in the same country must sell for the same price when their prices are expressed in terms of the same currency. C) in competitive markets free of transportation costs and official barrier to trade, identical goods sold in different countries must sell for the same price. D) identical goods sold in different countries must sell for the same price when their prices are expressed in terms of the same currency. E) in competitive markets free of official barrier to trade, identical goods are sold at the same price regardless of transportation costs.

A) in competitive markets free of transportation costs and official barriers to trade, identical goods sold in different countries must sell for the same price when their prices are expressed in terms of the same currency.

If the goods' money prices do not change, an appreciation of the dollar against the pound A) makes British sweaters cheaper in terms of American jeans. B) makes British sweaters more expensive in terms of American jeans. C) doesn't change the relative price of sweaters and jeans. D) makes American jeans cheaper in terms of British sweaters. E) makes British jeans more expensive in Britain.

A) makes British sweaters cheaper in terms of American jeans.

Investment is usually A) more variable than consumption. B) less variable than consumption. C) as variable as consumption. D) It is hard to tell from the data whether investment is more or less variable than consumption. E) a larger component of the GNP than consumption.

A) more variable than consumption.

For most macroeconomists A) national income accounts and national output accounts are equal to each other. B) national income accounts exceed national output accounts. C) national output accounts exceed national income accounts. D) it is impossible to tell whether national income accounts equal to national output accounts. E) national income accounts is much more important than national output accounts.

A) national income accounts and national output accounts are equal to each other.

28) If people expect relative PPP to hold A) the difference between the interest rates offered by dollar and euro deposits will equal the difference between the inflation rates expected, in the United States and Europe, respectively, over the relevant horizon. B) the difference between the interest rates offered by dollar and euro deposits will equal the difference between the inflation rates expected in Europe and the United States, respectively. C) the difference between the interest rates offered by dollar and euro deposits will equal the difference between the inflation rates expected, over the relevant horizon, in the United States and Europe, respectively, in the short run. D) the difference between the interest rates offered by dollar and euro deposits will be above the difference between the inflation rates expected, over the relevant horizon, in the United States and Europe, respectively. E) the difference between the interest rates offered by dollar and euro deposits will be below the difference between the inflation rates expected, over the relevant horizon, in the United States and Europe, respectively.

A) the difference between the interest rates offered by dollar and euro deposits will equal the difference between the inflation rates expected, in the United States and Europe, respectively, over the relevant horizon.

13) After a permanent increase in the money supply A) the exchange rate overshoots in the short run. B) the exchange rate overshoots in the long run. C) the exchange rate smoothly depreciates in the short run. D) the exchange rate smoothly appreciates in the short run. E) the exchange rate remains the same.

A) the exchange rate overshoots in the short run.

4) If there is an excess supply of money A) the interest rate falls. B) the interest rate rises. C) the real money supply shifts left to make an equilibrium. D) the real money supply shifts right to make an equilibrium. E) the interest rate stays constant, but consumer confidence falters.

A) the interest rate falls.

The action of arbitrage is A) the process of buying a currency cheap and selling it dear. B) the process of buying a currency dear and selling it cheap. C) the process of buying and selling currency at the same price. D) the process of selling currency at different prices in different markets. E) the process of buying a currency and holding onto it to take it off the market.

A) the process of buying a currency cheap and selling it dear.

A country's gross national product (GNP) is A) the value of all final goods and services produced by its factors of production and sold on the market in a given time period. B) the value of all intermediate goods and services produced by its factors of production and sold on the market in a given time period. C) the value of all final goods produced by its factors of production and sold on the market in a given time period. D) the value of all final goods and services produced by its factors of production and sold on the market. E) the value of all final goods and services produced by its factors of production, excluding land, and sold on the market in a given time period.

A) the value of all final goods and services produced by its factors of production and sold on the market in a given time period.

GDP is supposed to measure A) the volume of production within a country's borders. B) the volume of services generated within a country's borders. C) the volume of production of a country's output. D) GNP plus depreciation. E) net unilateral transfers from foreigners.

A) the volume of production within a country's borders.

Identify the following as debit or credit entries in the Balance of Payments

An increase in export expenditure: credit - current An increase in foreign assets held in the US: credit - finan An increase in income payments: debit - current An increase in official reserve assets: debit - financial account An increase in US assets held abroad: debit - financial

Which of the following is a correct prediction based on the PPP model of the exchange​ rates? A. An increase in the U.S. interest rates leads to depreciation of the dollar. B. An increase in the U.S. output leads to depreciation of the dollar. C. An increase in the U.S. money supply leads to a​ long-run appreciation of the dollar. D. An increase in the U.S. output will lead to the proportional increase in inflation rate.

A. An increase in the U.S. interest rates leads to depreciation of the dollar.

What externalities may arise in a low income country after trade barriers are​ removed?

A. More incentives exist to exploit certain​ low-wage sectors of the​ population, such as​ children, to gain a cost advantage. B. More incentives exist to exploit differences in environmental regulations to gain a cost advantage. C. More incentives to increase specialization to exploit comparative​ advantage, leading to bigger losses to less productive sectors of the economy.

Which of the following is a reason why the use of trade barriers to enforce labor or environmental standards may be less efficient than other​ measures?

A. Trade barriers are very​ costly, creating deadweight losses in consumption and production at home. B. The trade barriers may come to be exploited by special interest groups for ulterior motives. C. Without a coalition of​ countries, which are difficult to​ assemble, the trade barriers may be ineffectual. D. The imposition of trade barriers may make conditions worse rather than​ better, since some producers may move their facilities into the informal sector of the economy.

Letting Upper E Subscript $ divided by pound and q Subscript $ divided by pound ​denote, respectively, the nominal and real exchange rates between the U.S. dollar and the U.K.​ pound, which of the following accurately describes how these rates change when a permanent increase occurs in the U.K. real money demand​ function? A. Upper E Subscript $ divided by pound increases and q Subscript $ divided by pound is unaffected. B. both Upper E Subscript $ divided by pound and q Subscript $ divided by pound will decrease . C. Upper E Subscript $ divided by pound decreases and q Subscript $ divided by pound is unaffected. D. both Upper E Subscript $ divided by pound and q Subscript $ divided by pound will increase .

A. Upper E Subscript $ divided by pound increases and q Subscript $ divided by pound is unaffected.

When​ high-definition television​ (HDTV) was first considered a possibility in the United​ States, the U.S. government held a competition to select the technical standards that would be used nationwide. Why would the government see an advantage to setting one​ standard?

A. With a newly introduced product​ (such as​ HDTV) where the​ "best standards" are​ unknown, it may be wise to hold a competition to prevent locking into or adopting inferior standards. B. Shared standards permit manufacturers to capture economies of scale. C. The adoption of common standards allows states to work together and expand their commercial ties.

If the prices of identical commodity​ baskets, after conversion to a single​ currency, differ markedly across​ countries, it can be concluded that A. absolute PPP is way off the mark . B. relative PPP holds up less well than absolute PPP. C. absolute PPP holds up quite well . D. both versions of PPP are meaningful .

A. absolute PPP is way off the mark

Suppose that output demand and supply trends induce people to expect​ Home's currency to appreciate in real terms against the currency of​ Foreign, its key trading partner. This implies that the expected real interest rate should be A. higher in Foreign . B. falling in Foreign . C. higher in Home . D. unaffected.

A. higher in Foreign

standards vary across countries because

A. national laws and regulations are usually initially adopted for strictly domestic​ reasons, and these reasons obviously differ from country to country. B. economic conditions and living standards vary dramatically across​ countries, and standards undoubtedly reflect these underlying conditions. C. many standards reflect history and​ culture, and these clearly vary widely across countries

For trade to take place, a country must face a world relative price that is A. different from the relative price that would prevail in the absence of trade. B. more than the relative price that would prevail in the absence of trade. C. the same as the relative price that would prevail in the absence of trade. D. less than the relative price that would prevail in the absence of trade.

A. different from the relative price that would prevail in the absence of trade.

Movement of labor from a Foreign country to the domestic (Home) economy A. increases the marginal product of labor in Foreign. B. increases the marginal product of labor at Home. C. leaves the marginal product of land unchanged in both countries. D. occurs only is the marginal product of labor is higher in Foreign than at Home.

A. increases the marginal product of labor in Foreign.

As more labor is used, holding capital constant, A. the marginal product of labor decreases. B. the shape of the production function gets steeper from left to right. C. the marginal product of labor increase.

A. the marginal product of labor decreases.

Given PUS and YUS,

An increase in the European money supply causes the euro to depreciate against the dollar, but it does not disturb the U.S. money market equilibrium.

) How do we allocate statistical discrepancy among the current, capital, and financial accounts? A) We have no way of knowing exactly how to allocate this discrepancy .B) Depend on the degree of certainty by which we attribute to these accounts. C) Divide it evenly amongst the three accounts. D) Depend on the convention adopted by the specific financial institution. E) Statistical discrepancy signals human errors made when dealing with financial accounts

Answer A

A U.S. citizen buys a newly issued share of stock in England, paying for his order with a check, which the British company deposits in its own U.S. bank account in New York. How is this transaction accounted for in the balance of payments? A) financial account, U.S. asset export B) current account, U.S. service import C) current account, British good export D) financial account, British asset import E) financial account, U.S. asset import

Answer A

A country's current account A) balance equals the change in its net foreign wealth. B) balance equals thechange in its foreign wealth.' C) surplus equals the change in its foreign wealth. D) deficit equals the change in its foreign wealth. E) balance equals its GNP.Answer: A

Answer A

A country's gross national product (GNP) is A) the value of all final goods and services produced by its factors of production and sold on the market in a given time period. B) the value of all intermediate goods and services produced by its factors of production and sold on the market in a given time period. C) the value of all final goods produced by its factors of production and sold on the market in a given time period. D) the value of all final goods and services produced by its factors of production and sold on the market. E) the value of all final goods and services produced by its factors of production, excluding land, and sold on the market in a given time period.

Answer A

For most macroeconomists A) gross national income and gross national product are the same B) gross national income exceeds gross national product. C) gross national product exceeds gross national product. D) it is hard to tell whether gross national income equal gross national product. E) gross national product is much more important than gross national income.

Answer A

For most macroeconomists A) national income accounts and national output accounts are equal to each other. B) national income accounts exceed national output accounts .C) national output accounts exceed national income accounts. D) it is impossible to tell whether national income accounts equal to national output accounts. E) national income accounts is much more important than national output accounts

Answer A

GDP is supposed to measure A) the volume of production within a country's borders. B) the volume of services generated within a country's borders. C) the volume of production of a country's output. D) GNP plus depreciation. E) net unilateral transfers from foreigners

Answer A

Government purchases currently take up about A) 20 percent of U.S. GNP, and this share has not changed much since the late 1950s. B) 38 percent of U.S. GNP, and this share has not changed much since the late 1950s. C) 18 percent of U.S. GNP, and this share has been increasing since the late 1950s. D) 18 percent of U.S. GNP, and this share has been decreasing since the late 1950s. E) 25 percent of U.S. GNP, and this share has been decreasing since the late1950s.

Answer A

Government savings, , is equal to A) T -G. B) T + G. C) T = G. D) T + G -I. E) T -G = I

Answer A

Investment is usually A) more variable than consumption. B) less variable than consumption. C) as variable as consumption. D) It is hard to tell from the data whether investment is more or less variable than consumption. E) a larger component of the GNP than consumption.

Answer A

The CA is equal to A) Y -(C -I + G). B) Y + (C + I + G). C) Y -(C + I + G). D) Y -(C + I -G). E) Y + (C -I -G)

Answer A

The official settlements balance or balance of payments is the sum of A) the current account balance, the capital account balance, the non reserve portion of the financial account balance, the statistical discrepancy. B) the current account balance and the capital account balance. C) the current account balance, the capital account balance, the non reserve portion of the financial account balance. D) the current account balance and the non reserve portion of the financial account balance. E) the current account balance and the interest in all investments.

Answer A

Which one of the following expressions is the MOST accurate? A) CA = EX -IM B) CA = IM -EX C) CA = EX = IM D) CA = EX + IM E) CA -IM = EX

Answer A

Which one of the following statements is the MOST accurate? A) The sale of a used textbook does generate income for factors of production. B) The sale of a used textbook does not generate income for any factor of production. C) The sale of a used textbook sometimes does and sometimes does not generate income for factors of production. D) It is hard to tell whether a sale of a used textbook does or does not generate income for factors of production. E) The sale of a used textbook is a part of the GNP.

Answer B

GDP is different than GNP in that A) it accounts for net unilateral transfers. B) it does not account for indirect business taxes. C) it does not account for a country's production using services with foreign-owned capital. D) it accounts for depreciation. E) it is unhelpful when tracking national income.

Answer C

In 2006, the United States had A) a surplus in the current account. B) a balancein the current account .C) a deficit in the current account. D) From 2006 data, it is too difficult to determine whether a surplus or a deficit existed in the current account. E) a positive balance of net financial flows.

Answer C

In an open economy, private saving, , is equal to A) I -CA + (G -T). B) I + CA -(G -T). C) I + CA + (G -T). D) I -CA -(G -T). E) I + CA + (G + T).

Answer C

In open economies A) saving and investment are necessarily equal. B) as in a closed economy, saving and investment are not necessarily equal. C) saving and investment are not necessarily equal as they are in a closed economy. D) saving and investment are necessarily equal contrary to the case of a closed economy. E) investment always refers to the domestic stock market.

Answer C

Over the 1980s A) there is no question that a large increase in U.S. foreign assets did occur. B) there is a question whether a large decrease in U.S. foreign assets did occur. C) there is no question that a large decrease in U.S. foreign assets did occur. D) there is no question that there was almost no change inU.S. foreign assets. E) there is no question that rising exports exceeded U.S. foreign debt

Answer C

Ricardian equivalence argues that when the government cuts taxes and raises its deficit, A) consumers anticipate that they will face lower taxes later to pay for the resulting government debt. B) consumers anticipate that they will higher services from the government. C) consumers anticipate that they will face higher taxes later to pay for the resulting government debt. D) consumers anticipate it will affect their future taxes, in general in the direction of lowering future taxes. E) consumers anticipate that the low tax rates will continue.

Answer C

The sale of A) a used textbook does enter GNP .B) a used textbook does not enter GNP, but the sale of a used house does. C) both a used textbook and a used house do not enter GNP. D) a used house does not enter GNP, but the sale of a used book does. E) the GNP does not include saleof used items priced below $1000.

Answer C

The services British capital provides in Spain are a service export from Britain A) therefore they are subtracted from British GDP in calculating British GNP. B) therefore they are added to Spanish GDP in calculating Spanish GDP. C) therefore they are added to British GDP in calculating British GNP. D) therefore they are added to Spanish GNP in calculating Spanish GDP. E) therefore they are subtracted from Spanish GNP

Answer C

You travel to Paris and pay for a $100 dinner with your credit card. How is this accounted for in the balance of payments? A) current account, French service import B) current account, U.S. good export C) financial account, U.S. asset export D) financial account, U.S. asset import E) financial account, French asset export

Answer C

An example of how GNP accounts for services provided by foreign-owned capital (and GDP does not) is A) earnings of a Spanish factory with British owners counts only in Spain's GDP. B) earnings of a Spanish factory with British owners counts only in Britain's GNP. C) earnings of a Spanish factory counts in Spain's GNP but are part of Britain's GDP. D) earnings of a Spanish factory counts in Spain's GDP but are part of Britain's GNP. E) earnings of a Spanish factory counts in Spain's GNP but not in Britain's GDP or GNP.

Answer D

An open economy A) can save only by building up its capital stock. B) can save only by acquiring foreign wealth. C) cannot save either by building up its capital stock or by acquiring foreign wealth. D) can save either by building up its capital stock or by acquiring foreign wealth. E) can save by avoiding excessive imports.

Answer D

GNP equals GDP A) minus net receipts of factor income from the rest of the world. B) plus receipts of factor income from the rest of the world. C) minus receipts of factor income from the rest of the world. D) plus net receipts of factor income from the rest of the world.E) minus depreciation.

Answer D

Government purchases are defined as A) only goods purchased by federal, state, or local governments. B) all goods and services purchased by the federal government. C) all goods and services purchased by the federal or state government. D) all goods and services purchased by the federal, state, or local government. E) goods and services purchased from the government

Answer D

The United States issues a $10,000 debt forgiveness to Argentina. How is this accounted for in the balance of payments? A) financial account, U.S. asset import B) current account, Argentina transfer payment C) current account, U.S. service export D) financial account, U.S. asset export E) current account, Argentina good import

Answer D

The highest component of GNP is A) the current account. B) investment. C) government purchases. D) consumption. E) trade.

Answer D

Which of the following statements about the central bank is TRUE? A) Only the central bank may hold foreign reserves and intervene officially in exchange markets. B) Central banks have little power to alter macroeconomic conditions. C) Today, central banks' reserves consist largely of gold. D) The Federal Reserve holds only a small level of official reserve assets other than gold. E) Central banks never inject money into the economy

Answer D

An American buys a Japanese car, paying by writing a check on an account with a bank in New York. How would this be accounted for in the balance of payments? A) current account, a Japanese good import B) current account,a U.S. good import C) financial account, a U.S. asset import D) financial account, a U.S. asset export E) a current account as a U.S. good import and a financial account, a U.S. asset export

Answer E

Which of the following is TRUE? A) A country with a current account surplus is earning more from its exports than it spends on imports. B) A country could finance a current account deficit by using previously accumulated foreign wealth to pay for its imports. C) A country with a current account deficit must be increasing its net foreign debts by the amount of the deficit. D) We candescribe the current account surplus as the difference between income and absorption. E) All of the above are true of current account balances.

Answer E

Which one of the following statements is FALSE? A) The United States had accumulated substantial foreign wealth by the early 1980s. B) The 1980s witnessed a sustained current account deficit of proportions unprecedented in the twentieth century opened up. C) In 1987, the country became a net debtor to foreigners for the first time since World War I. D) U.S. foreign debt has continued to grow and now stands at 25 percent of GNP. E) The U.S. foreign debt was paid off in the 1990s, allowing the U.S. to attain a current account surplus. However, the deficit has returned in recent years

Answer E

What are the main aspects of economic life that macroeconomics analysis is most concerned with?

Answer: There are four main aspects: unemployment, saving, trade imbalances, and money and the price level.

What is the national income identity for a closed economy?

Answer: Y = C + I + G

What is the national income identity for an open economy?

Answer: Y = C + I + G + EX -IM

In the United States, (gross) investment has fluctuated between ________ of GNP in recent years. A) 2 and 12 percent B) 11 and 22 percent C) 22 and 32 percent D) 32 and 42 percent E) 42 and 52 percent

AnswerB

the IPP is equal​ to:

Assets owned outside the U.S. by U.S. citizens minus Assets owned inside the U.S. by foreign citizens.

A pollution haven is said to exist when A. the industrial countries choose an​ area, usually in the less developed​ world, to dispose of industrial wastes. B. an economic activity becomes concentrated in countries or regions having less strict environmental controls than elsewhere. C. a country fails to prosecute violators of its environmental regulations. D. a poorer region or country agrees to store pollutants in exchange for aid.

B

Floating exchange rates A) systematically lead to much larger but less frequent short-run deviations from the absolute PPP. B) systematically lead to much larger and more frequent short-run deviations from the relative PPP. C) systematically lead to much smaller and less frequent short-run deviations from the relative PPP. D) systematically lead to much smaller but more frequent short-run deviations from the relative PPP. E) systematically lead to much smaller and less frequent short-run deviations from the absolute PPP.

B

For main industrial countries such as Japan and the U.S. A) there is much less month-to-month variability of the exchange rate, suggesting that price levels are relatively sticky in the short run. B) there is much more month-to-month variability of the exchange rate, suggesting that price levels are relatively sticky in the short run. C) there is almost the same month-to-month variability of the exchange rate and price levels. D) it is hard to tell whether month-to-month variability of the exchange rate is similar to changes in price levels. E) there is much more month-to-month variability of the exchange rate, suggesting that price levels are relatively sticky in the long run.

B

For most macroeconomists, A. national output accounts exceed national income accounts. B. national income accounts and national output accounts are equal to each other. C. national income accounts exceed national output accounts. D. it is impossible to tell whether national income accounts equal to national output accounts. E. national income accounts is much more important than national output accounts.

B

GNP​ (Gross National​ Product) equals GDP plus A. indirect business taxes. B. net receipts of factor income from the rest of the world. C. a statistical discrepancy. D. the capital consumption allowance.

B

If firms in an industry are generating knowledge that other firms can use without paying for it, this industry is characterized by A) social costs that exceed private costs. B) social benefits that exceed private benefits. C) social costs that exceed social benefits. D) private benefits that exceed social benefits. E) social benefits that undermine private benefits.

B

If governments make trade policies based on national economic​ welfare, is the problem of trade warfare still represented by a​ "Prisoner's dilemma"​ game? What is the equilibrium solution to the game if governments formulate policy in this​ way? Would they ever choose the strategy of​ protectionism? A. It is still a​ Prisoner's dilemma​ game; however, now each​ country's dominant strategy would be to engage in free trade and there would be no incentive to switch to protectionism. B. In this​ case, it would no longer be a​ Prisoner's dilemma​ game; each​ country's dominant strategy would be to engage in free trade and there would be no incentive to switch to protectionism. C. It is still a​ Prisoner's dilemma​ game; however, now each​ country's dominant strategy would be to engage in free trade. This equilibrium is unstable as there would be an incentive for large countries to switch to protectionism to reap additional gains. D. In this​ case, it would no longer be a​ Prisoner's dilemma​ game; each​ country's dominant strategy would be to engage in free trade. ​ However, this equilibrium is unstable as there would be an incentive for large countries to switch to protectionism to reap additional gains.

B

If individuals are holding more money than they desire A) they will attempt to reduce their liquidity by using money to purchase goods. B) they will attempt to reduce their liquidity by using money to purchase interest-bearing assets. C) they will attempt to reduce their liquidity by converting real money holdings into nominal money holdings. D) they will keep their holdings constant.

B

If there is initially an A) excess demand for money, the interest rate will fall, and the supply of money it will rise. B) excess supply of money, the interest rate will fall, and if there is initially an excess demand, it will rise. C) excess supply of money, the interest rate will rise, and if there is initially an excess demand, it will fall. D) excess supply of money, the interest rate will fall, and if there is also an excess demand, it will fall rapidly. E) excess supply of money, the interest rate will rise, and if there is also an excess demand, it will rise rapidly.

B

In 1980 the United States announced an embargo on grain exports to the Soviet Union in response to the Soviet invasion of Afghanistan. This embargo was mainly resisted by A) U.S. grain consumers of bread. B) U.S. grain producers. C) foreign grain producers. D) U.S. communists. E) economists concerned with U.S. terms of trade.

B

In 1990 the United States imposed trade embargoes on Iraq's international trade. The negative effect on Iraq's consumer surplus would be greater the A) less elastic Iraq's demand schedule. B) more elastic Iraq's demand schedule. C) greater Iraq's dependence on foreign products. D) more inelastic Iraq's supply schedule. E) less elastic Iraq's labor force is.

B

In a world where the price level could adjust immediately to its new long-run level after a money supply increase A) The dollar interest rate would increase because prices would adjust immediately and prevent the money supply from rising. B) The dollar interest rate would fall because prices would adjust immediately and prevent the money supply from rising. C) The dollar interest rate would fall because prices would adjust immediately and prevent the money supply from decreasing. D) The dollar interest rate would decrease because prices would adjust immediately and prevent the money supply from decreasing. E) The dollar interest rate would fall because prices would not be able to prevent the money supply from rising.

B

In a world with money and bonds only A) it is not risky to hold money. B) it is risky to hold money. C) risk is an important factor in the demand for money. D) there is no relationship between risk and holding money. E) assets become meaningless.

B

It is argued that high-tech industries typically generate new technologies but cannot fully appropriate the commercial benefits associated with their inventions or discoveries. If this is true then in order to maximize a country's real income, the government should A) tax the high-tech firms. B) subsidize the high-tech firms. C) protect the high-tech firms. D) outsource high-tech production. E) discourage high-tech investments.

B

Michael Woodford says the following is an advantage of interest-rate instruments for central banks. A) Conduct monetary policy without inflation. B) Conduct monetary policy even if checking deposits pay interest at competitive rates. C) Conduct monetary policy without government approval. D) Conduct monetary policy with consumers in mind. E) Conduct monetary policy with workers in mind.

B

Shall a government be concerned about a large current account deficit or​ surplus? A. The government should worry about current account​ surplus, but not deficit B. Both current account surplus and deficit might not be sustainable in a long run and are thus a concern C. There is no economic reason for a government to worry about either a surplus or a deficit D. The government should worry about current account​ deficit, but not surplus.

B

The difference between nominal and real interest rates is that A) nominal interest rates are measured in terms of a country's output, while real interest rates are measured in monetary terms. B) nominal interest rates are measured in monetary terms, while real interest rates are measured in terms of a country's output. C) nominal interest rates can fluctuate, while real interest rates always remain fixed. D) real interest rates can fluctuate, while nominal interest rates always remain fixed. E) real interest rates are the same in every country, while nominal interest rates are different for every country.

B

The domestic market failure argument is a particular case of the theory of A) the optimum, or first-best. B) the second best. C) the third best. D) the sufficing principle. E) the efficiency case for free trade.

B

The economic analysis of international trade says that specialization and free trade A. is good for everyone in both developed and​ less-developed countries. B. increases welfare for a country as a​ whole, but can also create losers as well as winners by shifting the distribution of income within the country C. increases welfare not only for a country as a​ whole, but also for every citizen within the country D. increases welfare for developed​ countries, but not for​ less-developed countries.

B

The exchange rate between currencies depends on A) the interest rate that can be earned on deposits of those currencies. B) the interest rate that can be earned on deposits of those currencies and the expected future exchange rate. C) the expected future exchange rate. D) national output. E) the interest rate that can be earned on deposits of those countries and the national output.

B

The optimum tariff is most likely to apply to A) a small tariff imposed by a small country. B) a small tariff imposed by a large country. C) a large tariff imposed by a small country. D) a large tariff imposed by a large country. E) an ad valorem tariff on a small country.

B

The proposal that trade agreements should include a system which monitors worker conditions and make the results available to consumers in the rich importing country A) is consistent with the Invisible Hand paradigm. B) is consistent with the market failure approach. C) is consistent with the Ricardian theory of comparative advantage. D) is consistent with the scale economies approach to trade theory. E) is consistent with the principles laid out by the WTO.

B

The world trading system can be characterized as a combination of​ "levers" and​ "ratchets." In this analogy the ​"ratchetsratchets​" represent A. international tradeinternational trade negotiations that pushnegotiations that push trade liberalization forwardtrade liberalization forward. B. the implementationthe implementation of binding tariffsof binding tariffs which prevent backslidingwhich prevent backsliding. C. ​"carrotscarrots​" that enticethat entice desired behaviordesired behavior.

B

The​ "theory of the second​ best" states that A. free trade is only the​ "second best"​ policy, after the optimal tariff. B. in the presence of a market​ failure, government intervention may improve welfare. C. trade intervention is the best policy for dealing with domestic market imperfections. D. there is always an alternative solution if the first best is not feasible.

B

Today U.S. protectionism is concentrated in A) high-tech industries. B) labor-intensive industries. C) industries in which Japan has a comparative advantage. D) computer intensive industries. E) capital-intensive industries.

B

Trade theory suggests that Japan would gain from a subsidy the United States provides its grain farmers if the gains to Japanese consumers of wheat products more than offsets the losses to Japanese wheat farmers. This would occur as long as Japan A) is a net importer in bilateral trade flows with the United States. B) is a net importer of wheat. C) has a comparative advantage in wheat. D) has an absolute advantage in producing wheat. E) is involved in intra-industry trade with the United States.

B

Under U.S. commercial policy, which clause permits the modification of a trade liberalization agreement on a temporary basis if serious injury occurs to domestic producers as a result of the agreement? A) adjustment assistance clause B) escape clause C) most favored nation clause D) prohibitive tariff clause E) anti-dumping legislation

B

What is the main problem with imposing environmental​ standards? A. Consumers in developed countries are not willing to pay more for​ "clean" goods. B. They can cause potential export industries to shut down in poor countries. C. It is too costly to monitor. D. Countries cannot settle on a particular standard.

B

What is the main reason explaining why agriculture enjoys protective tariffs in the​ U.S.? A. Unfair competition from European agriculture. B. Producers​ (who gain) are well​ organized, while consumers​ (who lose) are not. C. Low wages in the agriculture sector would fall even further in the absence of protection. D. Environmental and health concerns force the government to restrict​ non-compliant imports.

B

What is the main reason for the​ anti-globalization movement in the late​ 1990s? A. Falling wages in​ low-wage countries due to trade B. Increasing manufacturing exports from​ low-wage countries C. Beggar-thy-neighbor policies D. Falling wages in domestic​ import-competing sectors.

B

Which of the following can help to explain why higher inflation may lead to currency appreciations? A) The interest rate is not the prime target of monetary policy. B) Most central banks adjust their policy interest rates expressly so as to keep inflation in check. C) Central banks increase the money supply leading to overshooting of the exchange rate. D) Inflation will increase the purchasing power of a currency. E) The world market does not adjust their currency trade to reflect inflation.

B

Which of the following statements is MOST accurate? A) In the output market, an increase in demand for U.S. output leads to an increase in the long-run nominal dollar/euro exchange rate. B) In the output market, an increase in the demand for European output leads to an increase in the long-run nominal dollar/euro exchange rate. C) In the output market, a decrease in demand for U.S. output leads to a decrease in the long-run nominal dollar/euro exchange rate. D) In the output market, an increase in the demand for European output leads to a decrease in the long-run nominal dollar/euro exchange rate. E) In the output market, an increase in the demand for European output leads to an increase in the long-run nominal euro/dollar exchange rate.

B

Which of the following statements is the MOST accurate? A) Relative PPP is not a reasonable approximation to the data. B) Relative PPP is sometimes a reasonable approximation to the data but often performs poorly. C) Relative PPP is sometimes a reasonable approximation to the data. D) PPP is sometimes a reasonable approximation to the data. E) PPP is sometimes a reasonable approximation to the data but usually performs poorly.

B

Which of the following statements is the MOST accurate? A) The prices of identical commodity baskets, when converted to a single currency, are the same across countries. B) The prices of identical commodity baskets, when converted to a single currency, differ substantially across countries. C) The prices of identical commodity baskets, when converted to a single currency, do not differ substantially across countries. D) The prices of identical commodity baskets, when converted to a single currency, are often the same across countries. E) The prices of identical commodity baskets, when converted to a single currency, are the same across countries more than 50% of the time.

B

Which one of the following statements is the most accurate? A. The sale of a used textbook does generate income for factors of production. B. The sale of a used textbook does not generate income for any factor of production. C. The sale of a used textbook is a part of the GNP. D. The sale of a used textbook sometimes does and sometimes does not generate income for factors of production. E. It is hard to tell whether the sale of a used textbook does or does not generate income for factors of production.

B

Which organization determines procedures for the settlement of international trade disputes? A) World Bank B) World Trade Organization C) International Monetary Organization D) International Bank for Reconstruction and Development E) The League of Nations

B

Who among the following list of people is an early 20th century economist from Yale University who wrote the book The Theory of Interest? A) Gustav Cassel B) Irving Fisher C) David Ricardo D) Paul Krugman E) Israel Kirzner

B

​France, in addition to its occasional stabs at strategic trade​ policy, pursues an active nationalist cultural ​policy, promoting French​ art, music,​ fashion, cuisine, and so on. This may be primarily a matter of attempting to preserve a national identity in an increasingly homogeneous​ world, but some French officials also defend this policy on economic grounds. In what sense could some features of such a policy be defended as a kind of strategic trade​ policy? A. There are no economic grounds for pursuing a nationalist cultural policy. B. Promoting French culture is a strategic trade policy in that it promotes tourism and industries related to French culture such as the textile and wine industries. C. Nationalist pride can encourage French workers and firms to produce high quality goods for export.

B

What is the exchange rate between the dollar and the British pound if a pair of American jeans costs 50 dollars in New York and 100 Pounds in London? A) 1.5 dollars per British pound B) 0.5 dollars per British pound C) 2.5 dollars per British pound D) 3.5 dollars per British pound E) 2 dollars per British pound

B) 0.5 dollars per British pound

In the United States, (gross) investment has fluctuated between ________ of GNP in recent years. A) 2 and 12 percent B) 11 and 22 percent C) 22 and 32 percent D) 32 and 42 percent E) 42 and 52 percent

B) 11 and 22 percent

What is the expected dollar rate of return on euro deposits if today's exchange rate is $1.10 per euro, next year's expected exchange rate is $1.166 per euro, the dollar interest rate is 10%, and the euro interest rate is 5%? A) 10% B) 11% C) -1% D) 0% E) 15%

B) 11%

14) Which of the following can help to explain why higher inflation may lead to currency appreciations? A) The interest rate is not the prime target of monetary policy. B) Most central banks adjust their policy interest rates expressly so as to keep inflation in check. C) Central banks increase the money supply leading to overshooting of the exchange rate. D) Inflation will increase the purchasing power of a currency. E) The world market does not adjust their currency trade to reflect inflation.

B) Most central banks adjust their policy interest rates expressly so as to keep inflation in check.

For open economies, A) S = I. B) S = I + CA. C) S = I - CA. D) S > I + CA. E) S < I + CA.

B) S = I + CA.

12) In a world where the price level could adjust immediately to its new long-run level after a money supply increase A) The dollar interest rate would increase because prices would adjust immediately and prevent the money supply from rising. B) The dollar interest rate would fall because prices would adjust immediately and prevent the money supply from rising. C) The dollar interest rate would fall because prices would adjust immediately and prevent the money supply from decreasing. D) The dollar interest rate would decrease because prices would adjust immediately and prevent the money supply from decreasing. E) The dollar interest rate would fall because prices would not be able to prevent the money supply from rising.

B) The dollar interest rate would fall because prices would adjust immediately and prevent the money supply from rising.

24) Which of the following statements is the MOST accurate? A) The prices of identical commodity baskets, when converted to a single currency, are the same across countries. B) The prices of identical commodity baskets, when converted to a single currency, differ substantially across countries. C) The prices of identical commodity baskets, when converted to a single currency, do not differ substantially across countries. D) The prices of identical commodity baskets, when converted to a single currency, are often the same across countries. E) The prices of identical commodity baskets, when converted to a single currency, are the same across countries more than 50% of the time.

B) The prices of identical commodity baskets, when converted to a single currency, differ substantially across countries.

Which one of the following statements is the MOST accurate? A) The sale of a used textbook does generate income for factors of production. B) The sale of a used textbook does not generate income for any factor of production. C) The sale of a used textbook sometimes does and sometimes does not generate income for factors of production. D) It is hard to tell whether a sale of a used textbook does or does not generate income for factors of production. E) The sale of a used textbook is a part of the GNP.

B) The sale of a used textbook does not generate income for any factor of production.

20) An increase in the world relative demand for U.S. output causes A) a short-run real depreciation of the dollar against the euro. B) a long-run real appreciation of the dollar against the euro. C) a long-run real depreciation of the dollar against the euro. D) a short-run real appreciation of the euro against the dollar. E) a long-run real appreciation of the euro against the dollar.

B) a long-run real appreciation of the dollar against the euro.

In a closed economy, national saving A) sometimes equals investment. B) always equals investment. C) is always less than investment. D) is always more than investment. E) is never equal to investment.

B) always equals investment.

Movements in GDP A) and GNP usually do not differ greatly. B) and GNP usually do not differ greatly, as a practical matter. C) and GNP usually do differ greatly. D) are usually smaller than those of GNP movements, in practice. E) are inversely proportional to movements in GNP.

B) and GNP usually do not differ greatly, as a practical matter.

The position of the United States current account balance in 2009 was A) lent over 6 percent of its GNP, resulting in a large current account surplus. B) borrowed over 9 percent of its GNP, leading to a large current account deficit. C) achieved a currant account balance of zero. D) borrowed over 10 percent of its GNP, leading to a large current account deficit. E) borrowed less then 5 percent of its GNP, leading to a large current account surplus.

B) borrowed over 9 percent of its GNP, leading to a large current account deficit.

A closed economy A) can save either by building up its capital stock or by acquiring foreign wealth. B) can save only by building up its capital stock. C) can save only by acquiring foreign wealth. D) cannot save either by building up its capital stock or by acquiring foreign wealth. E) can save by avoiding excessive imports.

B) can save only by building up its capital stock.

Movements in GDP A) differ greatly from movements in GNP. B) do not differ greatly from movements in GNP. C) are not allowed to differ at all from movements in GNP by definition. D) need to be inflation adjusted in order to match movements in GNP. E) are not relevant to an examination of national income.

B) do not differ greatly from movements in GNP.

Purchases of inventories by A) firms are not counted in investment spending. B) firms are also counted in investment spending. C) households are also counted in investment spending. D) households and Firms are also counted in investment spending. E) foreign consumers are counter in investment spending.

B) firms are also counted in investment spending.

3) A reduction in a country's money supply causes A) its currency to depreciate in the foreign exchange market. B) its currency to appreciate in the foreign exchange market. C) does not affect its currency in the foreign market. D) does affect its currency in the foreign market in an ambiguous manor. E) affects other countries currency in the foreign market.

B) its currency to appreciate in the foreign exchange market.

6) An increase in a country's money supply causes A) its currency to appreciate in the foreign exchange market while a reduction in the money supply causes its currency to depreciate. B) its currency to depreciate in the foreign exchange market while a reduction in the money supply causes its currency to appreciate. C) no effect on the values of it currency in international markets. D) its currency to depreciate in the foreign exchange market while a reduction in the money supply causes its currency to further depreciate. E) its currency to depreciate in the domestic market and appreciate in the foreign market.

B) its currency to depreciate in the foreign exchange market while a reduction in the money supply causes its currency to appreciate.

If the goods' money prices do not change, a depreciation of the dollar against the pound A) makes British sweaters cheaper in terms of American jeans. B) makes British sweaters more expensive in terms of American jeans. C) makes American jeans more expensive in terms of British sweaters. D) doesn't change the relative price of sweaters and jeans. E) makes British jeans more expensive in Britain.

B) makes British sweaters more expensive in terms of American jeans.

18) The difference between nominal and real interest rates is that A) nominal interest rates are measured in terms of a country's output, while real interest rates are measured in monetary terms. B) nominal interest rates are measured in monetary terms, while real interest rates are measured in terms of a country's output. C) nominal interest rates can fluctuate, while real interest rates always remain fixed. D) real interest rates can fluctuate, while nominal interest rates always remain fixed. E) real interest rates are the same in every country, while nominal interest rates are different for every country.

B) nominal interest rates are measured in monetary terms, while real interest rates are measured in terms of a country's output.

Government transfer payments like social security and unemployment benefits are A) included in government purchases. B) not included in government purchases. C) not included in government purchases, but they are included in the consumption component of GNP. D) not included in government purchases, but they are part of the investment component of GNP. E) included in government purchases but not in the GNP.

B) not included in government purchases.

In 1929, government purchases accounted for A) only 18.5 percent of U.S. GNP. B) only 8.5 percent of U.S. GNP. C) 28.5 percent of U.S. GNP. D) 38.5 percent of U.S. GNP. E) 48.5 percent of U.S. GNP.

B) only 8.5 percent of U.S. GNP.

What term means an explosive and seemingly uncontrollable inflation in which money loses value rapidly and may even go out of use? A) superinflation B) stagflation C) hyperinflation D) maginflation E) deflation

C

1) The exchange rate between currencies depends on A) the interest rate that can be earned on deposits of those currencies. B) the interest rate that can be earned on deposits of those currencies and the expected future exchange rate. C) the expected future exchange rate. D) national output. E) the interest rate that can be earned on deposits of those countries and the national output.

B) the interest rate that can be earned on deposits of those currencies and the expected future exchange rate.

15) For main industrial countries such as Japan and the U.S. A) there is much less month-to-month variability of the exchange rate, suggesting that price levels are relatively sticky in the short run. B) there is much more month-to-month variability of the exchange rate, suggesting that price levels are relatively sticky in the short run. C) there is almost the same month-to-month variability of the exchange rate and price levels. D) it is hard to tell whether month-to-month variability of the exchange rate is similar to changes in price levels. E) there is much more month-to-month variability of the exchange rate, suggesting that price levels are relatively sticky in the long run.

B) there is much more month-to-month variability of the exchange rate, suggesting that price levels are relatively sticky in the short run.

Some retailers in advanced countries sell products from developing countries with low wages but assure customers that these goods are produced under tolerable working conditions. Is demanding that kind of guarantee the same thing as putting a tariff on​ low-wage exports? Demanding the working conditions guarantee A. is the same thing as a tariff because both decrease import consumption. B. is different than a tariff because it increases consumer demand. C. is different than a tariff because it decreases market prices. D. is different than a tariff because it raises government revenue. E. is the same thing as a tariff because both increase production costs. Is there any way demanding a working conditions guarantee can benefit workers​ overseas? Demanding tolerable working conditions A. could benefit overseas workers if the demand for their labor decreases. B. could hurt overseas workers if their employers earn higher profits. C. will not affect overseas workers if retailers receive more revenue. D. could benefit overseas workers if they are paid higher wages.

B, D

In the real world where relative PPP fails to​ hold, suppose that U.S. inflation is expected to exceed European inflation left parenthesis pi Subscript font size decreased by 1 US Superscript e Baseline minus pi Subscript Upper E Superscript e Baseline right parenthesis by 6 percent for the foreseeable future.​ Furthermore, suppose that output demand and supply trends are widely expected to cause the dollar to decline against the euro in real terms at a rate of 1 percent per year. In this​ case, the international interest rate spread left parenthesis Upper R Subscript $ Baseline minus Upper R Subscript euro Baseline right parenthesis will actually be A. negative 5 percent. B. 7 percent. C. 5 percent. D. ​indeterminate, since expectations are unreliable.

B. 7 percent.

Which of the following correctly represents the formula for relative​ PPP? A. pi Subscript Upper E comma t ​= pi Subscript US comma t ​+ ​(Upper E Subscript $ divided by euro comma t minus Upper E Subscript $ divided by euro comma t minus 1 ​)/Upper E Subscript $ divided by euro comma t minus 1 . B. pi Subscript US comma t ​= pi Subscript Upper E comma t ​+ ​(Upper E Subscript $ divided by euro comma t minus Upper E Subscript $ divided by euro comma t minus 1 ​)/Upper E Subscript $ divided by euro comma t minus 1 . C. pi Subscript Upper E comma t ​= pi Subscript US comma t ​+ ​(Upper E Subscript $ divided by euro comma t plus 1 minus Upper E Subscript $ divided by euro comma t ​)/Upper E Subscript $ divided by euro comma t . D. pi Subscript US comma t ​= pi Subscript Upper E comma t ​+ ​(Upper E Subscript $ divided by euro comma t plus 1 minus Upper E Subscript $ divided by euro comma t​)/Upper E Subscript $ divided by euro comma t.

B. pi Subscript US comma t ​= pi Subscript Upper E comma t ​+ ​(Upper E Subscript $ divided by euro comma t minus Upper E Subscript $ divided by euro comma t minus 1 ​)/Upper E Subscript $ divided by euro comma t minus 1 .

Carefully follow the instructions above and only draw the required object. According to your​ diagram, the transfer from the Czech Republic to Poland means that A. the koruna has undergone a real appreciation against the zloty . B. the relative price of products from Poland has risen in terms of products from the Czech Republic . C. the koruna ​'s purchasing power within the borders of Poland has risen relative to its purchasing power within the Czech Republic . D. a smaller basket of goods from the Czech Republic is now required to purchase one basket from Poland .

B. the relative price of products from Poland has risen in terms of products from the Czech Republic . RD2 shift right

in 1986, the price of oil on world markets dropped sharply. Since the United States is an oil-importing country, this was widely regarded as good for the US economy. Yet in Texas and Louisiana, 1986 was a year of economic decline. Why? In Texas and Louisiana, 1986 was a year of economic decline because in these two states, A. a significant fraction of workers in the oil industry transferred to other sectors. B. Oil production was reduced. C. even though owners of capital in the oil industry benefitted, workers in the oil industry experienced high rates of unemployment.

B. Oil production was reduced.

If Money supply increases

Basically we have more money but the same amount of income, This would mean that the exchange rate would also increase depreciating the dollar against the foreign currency. (if theres is the same amount of income going into a higher money supply there is more of it meaning it is less valuable) Opposite affects if the euro money supply is increasing it would appreciate the dollar in relation to the euro.

A game-theory explanation of the paradox that even though all countries would benefit if each chose free trade, in fact each tends to follow protectionist policies is A) trade war. B) collective action. C) prisoner's dilemma. D) benefit-cost analysis. E) rent seeking.

C

A permanent increase in a country's money supply A) causes a more than proportional increase in its price level. B) causes a less than proportional increase in its price level. C) causes a proportional increase in its price level. D) leaves its price level constant in long-run equilibrium. E) causes an inversely proportional fall in its price level.

C

A trade policy designed to alleviate some domestic economic problem by exporting it to foreign countries is know as a(n) A) international dumping policy. B) countervailing tariff policy. C) beggar thy neighbor policy. D) trade adjustment assistance policy. E) redistribution quota policy.

C

In an open economy holding GNP and consumption spending constant and where private savings equals domestic​ investment, a government budget deficit must be matched by A. a positive difference between domestic exports and imports. B. a current account balance. C. a current account deficit. D. a current account surplus.

C

An economy's long-run equilibrium is A) the equilibrium that would occur if prices were perfectly flexible. B) the equilibrium that would occur if prices were perfectly flexible and always adjusted immediately. C) the equilibrium that would occur if prices were perfectly flexible and always adjusted immediately to preserve full employment. D) the equilibrium that would occur if prices were perfectly fixed to preserve full employment. E) the equilibrium that would occur if prices were perfectly fixed at the full employment point.

C

An issue never confronted effectively by GATT, but considered an important issue for WTO is that of A) the promotion of freer World trade. B) the promotion of freer World commodity trade. C) the promotion of freer World services trade. D) the lowering of tariff rates. E) the liberalization of trade.

C

Brander-Spencer analysis states that A. competition can be enhanced by international trade. B. monopoly profits can be eliminated by international trade. C. monopoly profits can be moved from one country to another by a government subsidy. D. monopoly profits can be eliminated by trade policies.

C

Capital gains and losses on a​ country's net foreign assets are not included in the national income measure of the current account. How would economic statisticians have to modify the national income identity Upper Y equals Upper C plus Upper I plus Upper G plus Upper X minus Upper M nbspY = C+I +G + X − M if they did wish to include such gains and losses as part of the definition of the current​ account? A. Y​ = C​ + I​ + G​ + X minus− M​ + Gross capital gain. B. Y​ = C​ + I​ + G​ + X minus− Mminus− Net capital gain. C. Y​ = C​ + I​ + G​ + X minus− M​ + Net capital gain. D. Y​ = C​ + I​ + G​ + X minus−M minus− Gross capital gain.

C

Criticisms of the Brander-Spencer model include all EXCEPT which of the following? A) the problem of insufficient information B) the problem of likely foreign retaliation C) the problem of harm to interests of consumers D) the problem of adverse effects of trade policy politics E) the problem of simultaneously causing harm to other industries

C

Do data on the U.S. official settlements balance give an accurate picture of the extent to which foreign central banks buy and sell dollars in currency​ markets? A. ​Yes, this account balance is equal to the total net sales of dollars by foreign central banks. B. ​Yes, this account balance is equal to the total net purchases of dollars by foreign central banks. C. ​No, this account provides only a partial picture because it shows a net value of all transactions. D. ​No, this account does not include sales or purchases of dollars by foreign central banks.

C

Faced with the evidence of poor working conditions and low wages in the border maquiladoras​, Economists A. argue that​ Mexico's generally high overall productivity offsets these conditions. B. shrug their shoulders and ignore the issue. C. argue that the poor conditions and low wages are actually improvements for the Mexican​ workers, and may be cited as gainsminus−fromminus−trade. D. agree that trade theory is thus proven hollow and internally inconsistent. E. argue that U.S. consumers should not consume lettuce.

C

For open economies, A. S = I. B. S = I - CA. C. S = I + CA. D. S > I + CA. E. S < I + CA.

C

GDP is different than GNP in that A. it accounts for depreciation. B. it does not account for indirect business taxes. C. it does not account for a country's production using services with foreign − owned capital. D. it is unhelpful when tracking national income. E. it accounts for net unilateral transfers.

C

GDP is supposed to measure A. the volume of production of a country's output. B. net unilateral transfers from foreigners. C. the volume of production within a country's borders. D. GNP plus depreciation. E. the volume of services generated within a country's borders.

C

GNP (Gross National Product) equals GDP plus A. indirect business taxes. B. the capital consumption allowance. C. net receipts of factor income from the rest of the world. D. a statistical discrepancy.

C

Government purchases are defined as A. goods and services purchased from the government. B. all goods and services purchased by the federal or state government. C. all goods and services purchased by the federal, state, or local government. D. only goods purchased by federal, state, or local governments. E. all goods and services purchased by the federal government.

C

How does the WTO justify its position on trade disputes that involve environmental​ issues? A. Imposing uniform environmental standards on all countries prevents any one country from gaining an unfair advantage. B. It requires developed countries to meet the same standards. C. It forces member countries to apply the same standards to imported goods that it applies to domestically produced goods. D. The WTO argues that it is not its place to impose environmental regulations on sovereign nations.

C

If the United States had its​ way, it would demand that Japan spend more money on basic research in science and less on applied research into industrial applications. Explain why in terms of the analysis of appropriability. A. Research targeting specific problems in an industry does not produce an externality that can be appropriated by other firms. Only basic research generates a technology externality. B. Research targeting problems in specific industries will lead to a competitive advantages for Japan that the U.S. cannot appropriate. C. The benefits to the United States of Japanese basic research would exceed the benefits from Japanese research targeted to specific problems in Japanese industries. D. All of the above.

C

In Zimbabwe, the government stopped the country's hyperinflation by A) reducing domestic monetary growth drastically. B) returning to a gold/silver currency standard. C) switching to foreign currencies. that are relatively stable. D) passing a law making price increases illegal. E) implementing a new currency based on diamonds.

C

In a closed economy, national saving A. is always more than investment. B. is always less than investment. C. always equals investment. D. sometimes equals investment. E. is never equal to investment.

C

In a closed​ economy, national saving A. sometimes equals investment. B. is never equal to investment. C. always equals investment. D. is always less than investment. E. is always more than investment.

C

In the long run A) exchange rates obey relative PPP when all disturbances occur in the output markets. B) exchange rates obey absolute PPP when all disturbances occur in the output markets. C) exchange rates are unlikely to obey relative PPP when all disturbances occur in the output markets. D) exchange rates are unlikely to obey relative PPP when all disturbances are monetary in nature. E) exchange rates obey absolute PPP when all disturbances are monetary in nature.

C

Labor standards in trade are typically opposed by most developing countries who believe that they will be used A) to further neo-imperialist colonial exploitation. B) to charge these countries with crimes against child-labor standards at the Hague. C) as a protectionist tool by import-competing producers in industrial countries. D) as a means of spreading U.S. Corporate Values and destroying local cultures. E) to hinder investment in foreign-based multinational corporations.

C

Low wages and poor working conditions in many U.S. trade partners A) prove that the gains-from-trade arguments of the Ricardian model are false. B) may be a fact of life, but economists don't care. C) are facts emphasized by U.S. labor in its contract negotiations. D) prove that the gains-from-trade arguments of the Ricardian model are true. E) prove that international trade is exploitative.

C

Many countries have​ value-added taxeslong dash—taxes that are paid by​ producers, but are intended to fall on consumers.​ (They're basically just an indirect way of imposing sales​ taxes.) Such​ value-added taxes are always accompanied by an equal tax on​ imports; such import taxes are considered legal​ because, like the​ value-added tax,​ they're really an indirect way of taxing all consumer purchases at the same rate. Compare this situation to the argument over carbon tariffs. Which of the following statements regarding carbon tariffs is​ true? A. A carbon tariff is put into place to encourage domestic firms to move their production to a pollution haven with lax environmental regulations. B. A carbon tariff would give an artificial advantage to domesticdomestic firms if it would be more costly for a domesticdomestic firm to reduce its carbon emissions than a foreignforeign firm. C. An objection over carbon tariffs is that they may be discriminating between domestic and foreign goods. D. By imposing a carbon tariff on​ imports, the overall level of pollution will be increased.

C

Some economists argue that conventional analysis of international trade policy underestimates the disruption to communities in the U.S. caused by rapid shifts in international trade. The reasons put forth to justify why impacts sometimes fall more heavily on certain communities​ include: A. uneven growth across industries in exports by foreign​ countries, the high geographic concentration of certain U.S. manufacturing​ industries, and the willingness of U.S. workers to relocate away from depressed regions B. even growth across industries in exports by foreign​ countries, the high geographic concentration of certain U.S. manufacturing​ industries, and the unwillingness of U.S. workers to relocate away from depressed regions C. uneven growth across industries in exports by foreign​ countries, the high geographic concentration of certain U.S. manufacturing​ industries, and the unwillingness of U.S. workers to relocate away from depressed regions D. uneven growth across industries in exports by foreign​ countries, the even distribution of U.S. manufacturing industries across regions of the​ country, and the unwillingness of U.S. workers to relocate away from depressed regions.

C

Spencer and Brander's model highlights the existence of A) aircraft industries. B) excess returns present in highly competitive markets. C) excess returns, or rents, available in non-competitive markets. D) the futility of government bureaucrats' attempts to build an airplane. E) natural advantages in foreign technology firms.

C

The Shipbreakers of Alang represent a perfect example of how a developing country can apply the principles of the Heckscher-Ohlin model, since A) shipbreaking is generally considered to be a capital-intensive operation and India, being a large country has much capital. B) shipbreaking is a labor-intensive operation in India, and India has many workers since it is such a large country. C) shipbreaking is a labor-intensive operation in India, and India's availability of capital per worker is less than that of its trade partners. D) shipbreaking is a capital-intensive operation elsewhere in the world, and therefore represents a case of a factor intensity reversal. E) India's climate lends itself to the work involved in shipbreaking.

C

The Smoot-Hawley Tariff Act of 1930 has generally been associated with A) falling tariffs. B) free trade. C) intensifying the worldwide depression. D) recovery from the worldwide depression. E) non-tariff barriers.

C

The WTO was established by the ________ of multilateral trade negotiations. A) Kennedy Round B) Tokyo Round C) Uruguay Round D) Dillon Round E) NAFTA Round

C

The WTO's intervention against clean air standards A) has earned it universal approval. B) was done in order to limit national sovereignty. C) has resulted in much criticism. D) has resulted in much criticism among professional economists. E) was championed in developing countries.

C

The World Trade Organization provides for all of the following EXCEPT A) the usage of the most favored nation clause. B) assistance in the settlement of trade disagreements. C) bilateral tariff reductions. D) multilateral tariff reductions. E) the prevention of nontariff interventions in trade.

C

The argument that strategic planning is not likely to be practical due to insufficient information means that A) because of trade secrets, the government does not know true cost relationships in any given industry. B) if the government had all the relevant information in a given industry then it could decide whether a subsidy would enhance the public's welfare. C) even if the government had all the relevant information in a given industry, it still could not decide whether a subsidy would enhance the public's welfare. D) due to recent cuts in the Department of the Census' sampling budgets, industry surveys are no longer reliable, so that there is no way to determine if a subsidy is in the public's interest. E) the government would need to employ its intelligence agencies in order to gain a complete understanding of the market.

C

The difference between Gross National Product​ (GNP) and National Income is a trivial amount. A. True B. False. National income equals GNP less depreciation less net unilateral transfers C. False. National income equals GNP less depreciation plus net unilateral transfers D. False. National income equals GNP plus depreciation less net unilateral transfers.

C

The efficiency case made for free trade is that as trade distortions such as tariffs are dismantled and removed A) government tariff revenue will decrease, and therefore national economic welfare will decrease. B) government tariff revenue will decrease, and therefore national economic welfare will increase. C) deadweight losses for producers and consumers will decrease, hence increasing national economic welfare. D) deadweight losses for producers and consumers will decrease, hence decreasing national economic welfare. E) government tariff revenue will increase, hence increasing national economic welfare.

C

The evidence usually cited to prove that globalization hurts workers in developing countries A) is inconclusive due to poor statistical design of the underlying samples. B) is inconclusive due to the poorly funded Central Statistical Office of Mexico. C) is inconclusive due to the ambiguous theoretical implications of the findings. D) is conclusive. E) does not take into account the Heckscher-Ohlin model.

C

The expected rate of change in the nominal dollar/euro exchange rate is best described as A) the expected rate of change in the real dollar/euro exchange rate minus the U.S.-Europe expected inflation difference. B) the expected rate of change in the real dollar/euro exchange rate plus the U.S.-Europe real interest rate difference. C) the expected rate of change in the real dollar/euro exchange rate plus the U.S.-Europe expected inflation difference. D) the expected rate of change in the real dollar/euro exchange rate minus the U.S.-Europe real interest rate difference. E) the expected rate of change in the real dollar/euro exchange rate plus the European expected inflation.

C

The fact that trade policy often imposes harm on large numbers of people, and benefits only a few may be explained by A) the lack of political involvement of the public. B) the power of advertisement. C) the problem of collective action. D) the basic impossibility of the democratic system to reach a fair solution. E) a cycle of political corruption.

C

The long run effects of money supply change A) ambiguous effect on the long-run values of the interest rate or real output, a proportional change in the price level's long-run value in the opposite direction. B) proportional effect on the long-run values of the interest rate or real output, a proportional change in the price level's long-run value in the same direction. C) no effect on the long-run values of the interest rate or real output, a proportional change in the price level's long-run value in the same direction. D) no effect on the long-run values of the interest rate or real output, no change in the price level's long-run value. E) ambiguous effect on the long-run values of the interest rate or real output, A disproportional change in the price level's long-run value in the same direction.

C

The notion that trade policy often imposes costs on large numbers of​ people, and benefits only a few is explained by A. the basic impossibility of the democratic system to reach a fair solution. B. the power of advertisement. C. the problem of collective action. D. the lack of political involvement of the public.

C

The official settlements balance or balance of payments is the sum of A. the current account balance and the non-reserve portion of the financial account balance. B. the current account balance and the capital account balance. C. the current account balance and the capital account balance, less the non-reserve portion of the financial account balance. D. the current account balance, the capital account balance, the non-reserve portion of the financial account balance, the statistical discrepancy. E. the current account balance and the interest in all investments.

C

Under PPP (and by the Fisher Effect), all else equal A) a rise in a country's expected inflation rate will eventually cause a more-than proportional rise in the interest rate that deposits of its currency offer in order to accommodate for the higher inflation. B) a fall in a country's expected inflation rate will eventually cause an equal rise in the interest rate that deposits of its currency offer. C) a rise in a country's expected inflation rate will eventually cause an equal rise in the interest rate that deposits of its currency offer. D) a rise in a country's expected inflation rate will eventually cause a less than proportional rise in the interest rate that deposits of its currency offer to accommodate the rise in expected inflation. E) a fall in a country's expected inflation rate will eventually cause an inversely proportional rise in the interest rate that deposits of its currency offer to accommodate the rise in expected inflation.

C

Under Purchasing Power Parity A) E$/E = PiUS/PiE. B) E$/E = PiE/PiUS. C) E$/E = PUS/PE. D) E$/E = PE/PES. E) E$/E = PiE + PiUS/PiE.

C

Under the monetary approach to exchange rate theory, money supply growth at a constant rate A) eventually results in ongoing price level deflation at the same rate, but changes in this long-run deflation rate do not affect the full-employment output level or the long-run relative prices of goods and services. B) eventually results in ongoing price level inflation at the same rate, but changes in this long-run inflation rate do affect the full-employment output level and the long-run relative prices of goods and services. C) eventually results in ongoing price level inflation at the same rate, but changes in this long-run inflation rate do not affect the full-employment output level or the long-run relative prices of goods and services. D) eventually results in ongoing price level inflation at the same rate, but changes in this long-run inflation rate do not affect the full-employment output level, only the long-run relative prices of goods and services. E) eventually results in ongoing price level deflation at the same rate, but changes in this long-run deflation rate do not affect the full-employment output level, only the long-run relative prices of goods and services.

C

What are the requirements of the​ GATT-WTO system? A. Import quotas cannot be reduced. B. Tariffs cannot exceed​ 1% C. Export subsidies are not allowed. D. Tariffs are not allowed.

C

What is the main critique against the WTO with respect to environmental​ protection? A. The​ WTO's stance on environmental protection supports protectionist trade policy. B. The WTO interferes with environmental policy of sovereign nations. C. It​ doesn't do enough to impose tougher environmental standards on developing countries. D. It imposes environmental standards on developing countries that they cannot afford.

C

What is the main problem with imposing labor standards to prevent child labor and poor working conditions in the exporting sector of a​ less-developed country? A. Countries cannot settle on a particular standard. B. It is too costly to monitor. C. ​Less-developed countries are opposed to them. D. Consumers in developed countries are not willing to pay more for goods produced in better conditions.

C

When one applies the Heckscher-Ohlin model of trade to the issue of trade-related income redistributions, one must conclude that North South trade, such as U.S.-Mexico trade, A) must help low skill workers on both sides of the border. B) is likely to hurt high-skilled workers in the U.S. C) is likely to involve higher overall national economic gains that will be greater than any harm done to low-skilled workers in the U.S. D) is likely to hurt low-skilled workers in Mexico. E) gives no advantage to the workers in either country.

C

Which of the following is NOT an account in the balance of​ payments? A. Capital account B. Financial account C. Future account D. Current account.

C

Which of the following is not part of the definition for Gross National Product? A. The market value... B. ...of all final goods and services... C. ...produced within a country's borders... D. ...within a given period of time.

C

Which of the following is not part of the definition for Gross National​ Product? A. The market value... B. ...of all final goods and services... C. ...produced within a​ country's borders... D. ...within a given period of time.

C

Which of the following statements is MOST accurate? A) The United States price level will place a relatively light weight on commodities produced and consumed in America, while the European price level will place a relatively heavy weight on commodities produced and consumed in Europe. B) The United States price level will place a relatively light weight on commodities produced and consumed in America, and the European price level will place a relatively light weight on commodities produced and consumed in Europe. C) The United States price level will place a relatively heavy weight on commodities produced and consumed in America, and the European price level will place a relatively heavy weight on commodities produced and consumed in Europe. D) The United States price level will place a relatively heavy weight on commodities produced and consumed in Europe, and the European price level will place a relatively heavy weight on commodities produced and consumed in America. E) The United States price level will place a relatively light weight on commodities produced and consumed in Europe, and the European price level will place a relatively heavy weight on commodities produced and consumed in America.

C

Which of the following statements is the MOST accurate about the Law of One Price on Scandinavian ferry lines? A) Due to menu costs, the Law of One Price does not hold. B) To avoid arbitrage opportunities, the Law of One Price must hold. C) Transaction costs of exchanging currency causes the Law of One Price to fail. D) Transportation costs between ferry lines leads to a violation of the Law of One Price. E) The physical distance allowed the Law of One Price to hold.

C

Which of the following statements is the MOST accurate? A) If PPP holds true, then the law of one price holds true for every commodity as long as the reference baskets used to reckon different countries' price levels are the same. B) If the law of one price holds true for every commodity, PPP must hold automatically. C) If the law of one price holds true for every commodity, PPP must automatically hold as long as the reference baskets used to reckon different countries' price levels are the same. D) If the law of one price does not hold true for every commodity, PPP cannot be true as long as the reference baskets used to reckon different countries' price levels are the same. E) If PPP holds true, then the law of one price must hold true automatically.

C

Which of the following statements is the MOST accurate? A) Relative PPP may be valid even when absolute PPP is not, provided the factors causing deviations from absolute PPP are more or less stable over different commodities space. B) Absolute PPP may be valid even when relative PPP is not, provided the factors causing deviations from relative PPP are more or less stable over time. C) Relative PPP may be valid even when absolute PPP is not, provided the factors causing deviations from absolute PPP are more or less stable over time. D) Relative PPP is not valid when absolute PPP is not. E) Relative PPP is only valid when absolute PPP is valid, providing the factors causing deviations from relative PPP are more or less stable over time.

C

Which of the following statements is the MOST accurate? In general, under the monetary approach to the exchange rate A) while the short-run interest rate does not depend on the absolute level of the money supply, continuing growth in the money supply eventually will affect the interest rate. B) while the long-run interest rate does depend on the absolute level of the money supply, continuing growth in the money supply do not affect the interest rate. C) while the long-run interest rate does not depend on the absolute level of the money supply, continuing growth in the money supply eventually will affect the interest rate. D) the long-run interest rate does not depend on the absolute level of the money supply, and thus continuing growth in the money supply will not affect the interest rate. E) while the short-run interest rate does not depend on the absolute level of the money supply, continuing decline in the money supply eventually will not affect the interest rate.

C

Which one of the countries below announces inflation targets? A) Japan B) U.S. C) Canada D) Mexico E) Nicaragua

C

Which one of the following expressions is the most accurate? A. CA − IM = EX B. CA = IM - EX C. CA = EX - IM D. CA = EX = IM E. CA = EX + IM

C

Which one of the following statements is the MOST accurate? A) Relative price changes could not lead to PPP violations even if trade were free and costless. B) Relative price changes could lead to PPP violations only if trade were free and costless. C) Relative price changes could lead to PPP violations even if trade were free and costless. D) Price changes could lead to PPP violations even if trade were free and costless. E) Price changes could not lead to PPP violations even if trade were free and costless.

C

Which one of the following statements is the MOST accurate? A) The purchasing power of any given country's currency will increase in countries where the prices of non-tradable goods rise. B) The purchasing power of any given country's currency will fall in countries where the prices of non-tradable goods fall. C) The purchasing power of any given country's currency will fall in countries where the prices of non-tradable goods rise. D) The purchasing power of any given country's currency will remain constant in countries where the prices of non-tradable goods rise. E) The purchasing power of any given country's currency will fall in countries where the prices of non-tradable goods remain constant.

C

Which one of the following statements is the MOST accurate? A) There is a lively academic debate over the possibility that seemingly sticky wages and prices are in reality quite fixed. B) There is a lively academic debate over the possibility that seemingly sticky wages and prices are in reality much more sticky than theory assumes. C) There is a lively academic debate over the possibility that seemingly sticky wages and prices are in reality quite flexible. D) There is no debate over the possibility that wages and prices are sticky in the long run. E) There is no debate over the possibility that wages and prices are sticky in the short run.

C

Working conditions for clothing workers in Bangladesh are very poor. If countries refuse to buy clothing from Bangladesh in order to encourage change, the effect is likely to be that A) firms will be forced to comply and workers will be better off. B) firms will refuse to comply, but workers will be better off. C) firms will try to comply and workers will be worse off. D) firms will try to comply and workers will be better off. E) regardless of how firms respond, workers will be better off.

C

You travel to Paris and pay for a $100 dinner with your credit card. How is this accounted for in the balance of payments? A. financial account, U.S. asset import B. current account, French service import C. financial account, U.S. asset export D. financial account, French asset export. E. current account, U.S. good export

C

In an open economy, private saving, , is equal to A) I - CA + (G - T). B) I + CA - (G - T). C) I + CA + (G - T). D) I - CA - (G - T). E) I + CA + (G + T).

C) I + CA + (G - T).

23) Which one of the following statements is the MOST accurate? A) Relative price changes could not lead to PPP violations even if trade were free and cost- less. B) Relative price changes could lead to PPP violations only if trade were free and costless. C) Relative price changes could lead to PPP violations even if trade were free and costless. D) Price changes could lead to PPP violations even if trade were free and costless. E) Price changes could not lead to PPP violations even if trade were free and costless.

C) Relative price changes could lead to PPP violations even if trade were free and costless.

19) Which of the following statements is MOST accurate? A) The United States price level will place a relatively light weight on commodities produced and consumed in America, while the European price level will place a relatively heavy weight on commodities produced and consumed in Europe. B) The United States price level will place a relatively light weight on commodities produced and consumed in America, and the European price level will place a relatively light weight on commodities produced and consumed in Europe. C) The United States price level will place a relatively heavy weight on commodities produced and consumed in America, and the European price level will place a relatively heavy weight on commodities produced and consumed in Europe. D) The United States price level will place a relatively heavy weight on commodities produced and consumed in Europe, and the European price level will place a relatively heavy weight on commodities produced and consumed in America. E) The United States price level will place a relatively light weight on commodities produced and consumed in Europe, and the European price level will place a relatively heavy weight on commodities produced and consumed in America.

C) The United States price level will place a relatively heavy weight on commodities produced and consumed in America, and the European price level will place a relatively heavy weight on commodities produced and consumed in Europe.

In 2006, the United States had A) a surplus in the current account. B) a balance in the current account. C) a deficit in the current account. D) From 2006 data, it is too difficult to determine whether a surplus or a deficit existed in the current account. E) a positive balance of net financial flows.

C) a deficit in the current account.

The sale of A) a used textbook does enter GNP. B) a used textbook does not enter GNP, but the sale of a used house does. C) both a used textbook and a used house do not enter GNP. D) a used house does not enter GNP, but the sale of a used book does. E) the GNP does not include sale of used items priced below $1000.

C) both a used textbook and a used house do not enter GNP.

Ricardian equivalence argues that when the government cuts taxes and raises its deficit, A) consumers anticipate that they will face lower taxes later to pay for the resulting government debt. B) consumers anticipate that they will higher services from the government. C) consumers anticipate that they will face higher taxes later to pay for the resulting government debt. D) consumers anticipate it will affect their future taxes, in general in the direction of lowering future taxes. E) consumers anticipate that the low tax rates will continue.

C) consumers anticipate that they will face higher taxes later to pay for the resulting government debt.

Which of the following type of funds cater to wealthy individuals, are not bound by government regulations, and are actively traded in foreign exchange markets? A) pension funds B) mutual funds C) hedge funds D) exchange funds

C) hedge funds

GDP is different than GNP in that A) it accounts for net unilateral transfers. B) it does not account for indirect business taxes. C) it does not account for a country's production using services with foreign-owned capital. D) it accounts for depreciation. E) it is unhelpful when tracking national income.

C) it does not account for a country's production using services with foreign-owned capital.

9) The long run effects of money supply change A) ambiguous effect on the long-run values of the interest rate or real output, a proportional change in the price level's long-run value in the opposite direction. B) proportional effect on the long-run values of the interest rate or real output, a proportional change in the price level's long-run value in the same direction. C) no effect on the long-run values of the interest rate or real output, a proportional change in the price level's long-run value in the same direction. D) no effect on the long-run values of the interest rate or real output, no change in the price level's long-run value. E) ambiguous effect on the long-run values of the interest rate or real output, A disproportional change in the price level's long-run value in the same direction.

C) no effect on the long-run values of the interest rate or real output, a proportional change in the price level's long-run value in the same direction.

In open economies A) saving and investment are necessarily equal. B) as in a closed economy, saving and investment are not necessarily equal. C) saving and investment are not necessarily equal as they are in a closed economy. D) saving and investment are necessarily equal contrary to the case of a closed economy. E) investment always refers to the domestic stock market.

C) saving and investment are not necessarily equal as they are in a closed economy.

17) The expected rate of change in the nominal dollar/euro exchange rate is best described as A) the expected rate of change in the real dollar/euro exchange rate minus the U.S.-Europe expected inflation difference. B) the expected rate of change in the real dollar/euro exchange rate plus the U.S.-Europe real interest rate difference. C) the expected rate of change in the real dollar/euro exchange rate plus the U.S.-Europe expected inflation difference. D) the expected rate of change in the real dollar/euro exchange rate minus the U.S.-Europe real interest rate difference. E) the expected rate of change in the real dollar/euro exchange rate plus the European expected inflation.

C) the expected rate of change in the real dollar/euro exchange rate plus the U.S.-Europe expected inflation difference.

Over the 1980s A) there is no question that a large increase in U.S. foreign assets did occur. B) there is a question whether a large decrease in U.S. foreign assets did occur. C) there is no question that a large decrease in U.S. foreign assets did occur. D) there is no question that there was almost no change in U.S. foreign assets. E) there is no question that rising exports exceeded U.S. foreign debt.

C) there is no question that a large decrease in U.S. foreign assets did occur.

Assume a specific factors economy produced two goods, cloth and food, and that when representing the output of this economy graphically, cloth is on the x-axis and food is on the y-axis. When the price of cloth increase by 5% and the price of food increases by 5%. A. the output of food falls. B. the output of cloths rises. C. the real incomes of landowners remain the same. D. labor shifts from the food sector to the clothes sector.

C. the real incomes of landowners remain the same.

Based on purchasing power parity​ (PPP), which of the​ following, all else being​ equal, could lead to a​ long-run real appreciation of the U.S.​ dollar? A. An increase in the price of oil that reduces the world demand for American cars. B. A permanent increase in the rate with which the Fed expands money supply. C. A rise in the growth rate of the U.S. GDP. D. ​Long-run nominal depreciation of the dollar.

C. A rise in the growth rate of the U.S. GDP.

The monetary approach to the exchange rate predicts that the dollar will depreciate in the long run​ if, ceteris paribus​, A. US money supply falls or Eurpoean money supply rises B. European output falls or US output rises C. US interest rate rises or European interest rate falls D. US interest rate falls and US money supply falls

C. US interest rate rises or European interest rate falls

When opening up to trade, an economy A. exports and import the good whose relative price has decreased. B. A. exports and import the good whose relative price has increase. C. exports the good whose relative price has increased and imports the god whose relative price has decreased. D. exports the good whose relative price has decreased and imports the good whose relative price has increased.

C. exports the good whose relative price has increased and imports the god whose relative price has decreased.

According to the one-good model, international mobility labor A. does not fully equalize real wages across countries. B. makes everyone in the world better off. C. increase the world's output. D. does not affect the marginal output of land.

C. increase the world's output.

Trade benefits A. all factors in the economy. B. the factor that is specific to the import - competing sectors. C. the factor that is specific to the export sector of each economy. D. mobile factors.

C. the factor that is specific to the export sector of each economy.

4) In the Brander-Spencer model the subsidy raises profits by more than the subsidy because of A) the "multiplier" effect of government expenditures. B) the military-industrial complex. C) the forward and backward linkage effects of certain industries. D) the deterrent effect of the subsidy on foreign competition. E) the economies of scale once the company enters the market.

D

A U.S. citizen buys a newly issued share of stock in England, paying for his order with a check, which the British company deposits in its own U.S. bank account in New York. How is this transaction accounted for in the balance of payments? A. current account, British good export B. financial account, U.S. asset export C. financial account, British asset import D. financial account, U.S. asset import E. current account, U.S. service import

D

A country's gross national product (GNP) is A. the value of all intermediate goods and services produced by its factors of production and sold on the market in a given time period. B. the value of all final goods and services produced by its factors of production and sold on the market. C. the value of all final goods and services produced by its factors of production, excluding land, and sold on the market in a given time period. D. the value of all final goods and services produced by its factors of production and sold on the market in a given time period. E. the value of all final goods produced by its factors of production and sold on the market in a given time period.

D

Capital gains and losses on a country's net foreign assets are not included in the national income measure of the current account. How would economic statisticians have to modify the national income identity Y = C + I + G + X - M if they did wish to include such gains and losses as part of the definition of the current account? A. Y = C + I + G + X - M - Net capital gain. B. Y = C + I + G + X - M + Gross capital gain. C. Y = C + I + G + X - M - Gross capital gain. D. Y = C + I + G + X - M + Net capital gain.

D

Changes in the money supply growth rate A) are neutral in the short run. B) need not be neutral in the short run. C) are neutral in the long run. D) need not be neutral in the long run. E) affect the real output of the economy.

D

Faced with the evidence of poor working conditions and low wages in the border maquiladoras, economists A) shrug their shoulders and ignore the issue. B) agree that trade theory is thus proven hollow and internally inconsistent. C) argue that U.S. consumers should not consume lettuce. D) argue that the poor conditions and low wages are actually improvements for the Mexican workers, and may be cited as gains-from-trade. E) argue that Mexico's generally high overall productivity offsets these conditions.

D

For most developing countries A) productivity is high among domestic workers. B) population growth and illiteracy rates are low. C) saving and investment levels are high. D) agricultural goods and raw materials constitute a high proportion of domestic output. E) pollution emissions are relatively low.

D

GNP accounts avoid double counting by including only the value of final goods and services sold on the market. Should the measure of imports and exports used in the GNP accounts therefore be defined to include only imports and exports of final goods and services received from and sold to other​ countries? A. Total value of imports should be​ included, but only the value of exports of final goods. B. ​Yes, only the value of final goods that are exported or imported should be included. C. Total value of exports should be​ included, but only the value of imports of final goods. D. ​No, total values and imports and exports should be included in the calculation of the GNP.

D

Give an intuitive explanation for the optimal tariff argument. A. In a small​ country, a tariff can unfavorably shift the terms of trade such that the welfare loss exceeds the tariff revenue. B. In a small​ country, a tariff can favorably shift the terms of trade such that the tariff revenue exceeds the welfare loss. C. In a large​ country, a tariff can unfavorably shift the terms of trade such that the tariff revenue exceeds the welfare loss. D. In a large​ country, a tariff can favorably shift the terms of trade such that the tariff revenue exceeds the welfare loss.

D

Given PUS and YUS A) An increase in the European money supply causes the euro to appreciate against the dollar, but it does not disturb the U.S. money market equilibrium. B) An increase in the European money supply causes the euro to appreciate against the dollar, and it creates excess demand for dollars in the U.S. money market. C) An increase in the European money supply causes the euro to depreciate against the dollar, and it creates excess demand for dollars in the U.S. money market. D) An increase in the European money supply causes the euro to depreciate against the dollar, but it does not disturb the U.S. money market equilibrium. E) An increase in the European money supply causes the euro to depreciate against the dollar, and disturbing the U.S. money market equilibrium.

D

Government savings, Sg is equal to A. T + G. B. T + G -I. C. T - G = I. D. T - G. E. T = G.

D

Gross National Product represents the sum of the following expenditure​ categories: A. ​consumption, investment, tax​ collections, and the current account balance. B. ​consumption, investment, government​ purchases, and the capital account balance. C. ​savings, investment, tax​ collections, and government purchases. D. ​consumption, investment, government​ purchases, and the current account balance.

D

In 2002 some European countries threatened to deny permission to land airplanes for airlines that enjoy a government subsidy. The countries claimed that the threat was targeted against​ Swissair, which was bailed out by the Swiss government. ​ However, it coincided with the U.S. imposing a tariff on steel imports and U.S. government compensation to the airlines for their losses due to the September 11th attack and increased security. Such a threat by the Europeans is an example A. policies not allowed by WTO B. the​ Brander-Spencer argument C. a desirable​ second-best outcome D. a trade policy that could lead to a trade war.

D

In January 2013, the world's cheapest Big Macs were sold in A) the Philippines. B) Russia. C) China. D) Malysia. E) the Czech Republic.

D

In a classic paper, Columbia University economist Phillip Cagan drew the line between inflation and hyperinflation at an inflation rate of A) more than 120 percent per year. B) more than 100 percent per year. C) more than 200 percent per year. D) more than 12,000 percent per year. E) more than 1,000 percent per year.

D

In an open economy holding GNP and consumption spending constant and where private savings equals domestic investment, a government budget deficit must be matched by A. a current account surplus. B. a current account balance. C. a positive difference between domestic exports and imports. D. a current account deficit.

D

In effect, the U.S. does subsidize high-tech firms by subsidizing R&D. This is done through A) the budget of the Department of Education. B) systematic protection through the levying of tariffs. C) systematic protection through the establishment of NTBs. D) relatively accelerated "depreciation" of R&D investment in the Federal tax codes. E) subsidies for high-tech firms.

D

In the second half of the 1990s a rapidly growing movement focused on the harm caused by international trade to A) land owners in poor countries. B) capital owners in rich industrialized countries. C) land owners in rich industrialized countries. D) production workers in both rich and poor countries. E) terms of trade in developing countries.

D

In today's world markets, poor developing countries tend to rely primarily on exports of A) agricultural products. B) primary products. C) mineral products. D) manufactured products. E) high-tech products.

D

Individuals base their demand for an asset on A) the expected return the asset offers compared with the returns offered by other assets. B) the riskiness of the asset's expected return. C) the asset's liquidity. D) the expected return, how risky that expected return is, and the asset's liquidity. E) the aesthetic qualities of the asset.

D

Inflation targeting was initiated by which central bank in 1989? A) U.S. B) Japan C) Canada D) New Zealand E) U.K.

D

It's widely believed that​ self-driving vehicles will become commonplace in the fairly near future and that their growth will be fast for years to come.​ Doesn't this mean that the United States should have policies designed to ensure that we are a leader in the​ self-driving car​ industry? Should the U.S. subsidize the domestic​ self-driving car​ industry? The U.S. should A. offer subsidies if the U.S. naturally has a comparative advantage in producing​ self-driving vehicles. B. offer subsidies if the subsidies would promote perfect competition. C. not offer subsidies if the subsidies would lead to technological spillovers. D. offeroffer subsidies if the subsidies wouldwould generate returns from market power in excess of subsidy costs. E. not offer subsidies if the subsidies would lead to excess returns above normal market levels.

D

Money includes A) currency. B) checking deposits held by households and firms. C) deposits in the foreign exchange markets. D) currency and checking deposits held by households and firms. E) futures and deposits in the foreign exchange market.

D

Money serves as all of the following EXCEPT A) a medium of exchange. B) a unit of account. C) a store of value. D) a symbol that is made of or can be redeemed for a fixed amount of precious metal. E) a highly liquid asset.

D

The Brander-Spencer model identified market failure in certain industries due to A) unfair competition. B) wildcat destructive competition. C) environmental negative externalities associated with pollution. D) limited competition. E) lack of excess returns.

D

The European Union​ (EU) began in 1957 as a customs union among six​ nations, and has widened and deepened the economic integration of Europe up until the Brexit decision in 2016. The reason for Brexitlong dash—British exit from the European​ Union, which was decided upon narrowly through a referendum held in that countrylong dash—is most likely related to A. the higher welfare benefits for British citizens that would arise from imposing large tariffs on imports from other EU member nations. B. the frustration that EU member​ nations, for purely political​ reasons, have not yet moved to reduce tariffs and export subsidies in agriculture to realize all the gains possible from free trade. C. domestic market failures that cause the level of social welfare with free trade to be below the level that would result with a​ welfare-maximizing tariff. D. the perception in Britain that the inward migration of poorer EU citizens from the former Communist countries in Eastern Europe have resulted in job losses for​ native-born British citizens and put a strain on public services.

D

The General Agreement on Tariffs and Trade and the World Trade Organization have resulted in A) the establishment of universal trade adjustment assistance policies. B) the establishment of the European Union. C) the reciprocal trade clause. D) reductions in trade barriers via multilateral negotiations. E) the total protection of all intellectual property rights.

D

The Shipbreakers of Alang arouse the ire of Greenpeace because of A) India's non-repentant nuclear stance. B) India's import-competing industrialization policies. C) the difficulty of avoiding ship accidents between Greenpeace's sailboat and the reconstructed Container ships of Alang. D) the large amount of pollution associated with the operations at Alang. E) their competition with capital-intensive industries.

D

The World Trade Organization (WTO) was organized as a successor to the A) IMF. B) UN. C) UNCTAD. D) GATT. E) the World Bank.

D

The aggregate money demand depends on A) the interest rate. B) the price level. C) real national income. D) the interest rate, price level, and real national income. E) the price level and the liquidity of the asset.

D

The aggregate real money demand schedule L(R,Y) A) slopes upward because a fall in the interest rate raises the desired real money holdings of each household and firm in the economy. B) slopes downward because a fall in the interest rate reduces the desired real money holdings of each household and firm in the economy. C) has a zero slope because a fall in the interest rate keeps constant the desired real money holdings of each household and firm in the economy. D) slopes downward because a fall in the interest rate raises the desired real money holdings of each household and firm in the economy. E) slopes downward because a rise in the interest rate makes consumers less focused on the liquidity of their assets.

D

The authors of the text believe that A) second-best policy is worse than optimal policy. B) special interest groups generally enhance national welfare. C) national welfare is likely to be enhanced by the imposition of an optimal tariff. D) market failure arguments tend to support free-trade policy. E) there is no such thing as national welfare.

D

The difficulty of ascertaining the right second-best trade policy to follow A) reinforces support for the third-best policy approach. B) reinforces support for increasing research capabilities of government agencies. C) reinforces support for abandoning trade policy as an option. D) reinforces support for free-trade options. E) reinforced support for the domestic market failure argument.

D

The efficiency case made for free trade is that as trade distortions are eliminated A. tariff revenue will​ decrease, and therefore national economic welfare will increase. B. tariff revenue will​ decrease, and therefore national economic welfare will decrease. C. efficiency losses for producers and consumers will​ decrease, hence decreasing national economic welfare. D. efficiency losses for producers and consumers will​ decrease, hence increasing national economic welfare.

D

The existence of marginal social benefits which are not marginal benefits for the industry producing the import substitutes A) is an argument supporting free trade and non-governmental involvement. B) is an argument supporting the use of an optimum tariff. C) is an argument supporting the use of market failures as a trade-policy strategy. D) is an argument rejecting free trade and supporting governmental involvement. E) is an argument rejecting the domestic market failure concept.

D

The expected real interest rate (re) in terms of the nominal interest rate (R) and the expected inflation rate (πe) is given by A) re = πe + R. B) re = 2πe + R2. C) re = πe + R2. D) re = R - πe. E) re = R2 - πe.

D

The fact that articles of clothing sold in Walmart are produced by very poorly paid workers in Honduras, is a fact that if taken into account A) would prove to economists that the Ricardian model of comparative advantage is false. B) would prove to economists that the equal-value in trade concept summed up in the trade triangles is incorrect. C) proves to economists that trade is a negative sum game. D) proves to the Anti-Globalization Movement that trade is a negative sum game. E) proves that corporations are exempt from labor standards.

D

The invocation of beggar-thy-neighbor arguments with respect to industrial policies A) strengthens the argument for subsidies. B) makes sense if the international Keynesian multipliers exceed unity. C) applies only to rich countries most of whose trade partners are very poor countries. D) weakens the argument for subsidies. E) does not apply to rich countries who can influence relative world prices.

D

The largest component of GNP is, A. investment. B. government purchases. C. trade. D. consumption. E. the current account.

D

The median voter model A) works well in the area of trade policy. B) is not intuitively reasonable. C) tends to result in biased tariff rates. D) does not work well in the area of trade policy. E) is not widely practiced in the United States.

D

The monetary approach makes the general prediction that A) the exchange rate, which is the relative price of American and European money, is fully determined in the long run by the relative supplies of those monies. B) the exchange rate, which is the relative price of American and European money, is fully determined in the short run by the relative supplies of those monies and the relative demands for them. C) the exchange rate, which is the relative price of American and European money, is fully determined in the short run and long run by the relative supplies of those monies and the relative demands for them. D) the exchange rate, which is the relative price of American and European money, is fully determined in the long run by the relative supplies of those monies and the relative demands for them. E) the money supply in the U.S. will adjust to European monetary equilibrium.

D

The political wisdom of choosing a tariff acceptable to the median U.S. voter is A) a good example of the principle of the second best. B) a good example of the way in which actual tariff policies are determined. C) a good example of the principle of political negotiation. D) not evident in actual tariff determination. E) usually evident in actual tariff determination.

D

The quantitative importance of U.S. protection of the domestic clothing industry is best explained by the fact that A) this industry is an important employer of highly skilled labor. B) this industry is an important employer of low skilled labor. C) most of the exporters of clothing into the U.S. are poor countries. D) this industry is a politically well organized sector in the U.S. E) the technology involved is very advanced.

D

The reason protectionism remains strong in the United States is that A) economists can produce any result they are hired to produce. B) economists cannot persuade the general public that free trade is beneficial. C) economists do not really understand how the real world works. D) the losses associated with protectionism are diffuse, making lobbying by the public impractical. E) economists cannot agree on trade policy recommendations

D

The shipbreakers of Alang are A) a metaphysical representation of the WTO, deriving from Edgar Rice Burroughs' Princess of Mars. B) an early version of the Russian Ice-breaker of the Dnieper-Alang class. C) a capital-intensive industry. D) competing with pollution-producing industries in countries outside of India. E) doing environmentally conscious work.

D

The simple model of competition among political parties long used by political scientists tends to lead to the practical solution of selecting the A) optimal tariff. B) prohibitive tariff. C) zero (free-trade) tariff. D) the tariff rate favored by the median voter. E) the tariff rate supported by exporters.

D

The world trading system combines negotiated agreements that promote trade liberalization called ________ with binding agreements called ________ that block tariff increases. A) fiscal policies; monetary policies B) truces; aggressions C) free trade; enforcement contracts D) levers; ratchets E) wheels; walls

D

What is the relationship between GATT and​ WTO? A. GATT is a more formal organization with larger scope B. WTO is a new name for GATT C. GATT is a new name for WTO D. WTO is a more formal organization with larger scope.

D

When Japan's MITI (Ministry of International Trade and Industry) focused resources on the semiconductor industry, and in particular on Random Access Memory (RAM), it was viewed as a typically successful Japanese foray into a new dynamic strategic sector. The results, as viewed by the late 1990s A) justified this view. B) led to similar structuring of industrial policy in the U.S. C) lent support to the Brander-Spencer model. D) helped shift the focus of economists away from Japanese-style industrial policy. E) propelled Japan into the leading country in high-tech manufacturing

D

Which of the following are theories meant to explain "Why Price Levels are Lower in Poorer Countries"? A) only Bhagwati-Kravis-Lipsey B) only Balassa-Samuelson C) only Goldberg-Knetter D) Bhagwati-Kravis-Lipsey and Balassa-Samuelson E) Bhagwati-Kravis-Lipsey and Goldberg-Knetter

D

Which of the following is NOT an argument to support free​ trade? A. Free trade leads to efficient allocation of resources B. Free trade limits the influence of​ special-interest groups C. Free trade allows firms to exploit economies of scale D. Free trade is always​ welfare-improving because those who gain can compensate those who lose.

D

Which of the following statements is the MOST accurate? A) Absolute PPP does not imply relative PPP. B) Relative PPP implies absolute PPP. C) There is no causality relation between the two. D) Absolute PPP implies relative PPP. E) Absolute PPP is inversely related to relative PPP.

D

Which of the following statements is the MOST accurate? A) In the long run, national price levels play a minor role in determining both interest rates and the relative prices at which countries' products are traded. B) In the long run, national price levels play a key role only in determining interest rates. C) In the long run, national price levels play a key role only in determining the relative prices at which countries' products are traded. D) In the long run, national price levels play a key role in determining both interest rates and the relative prices at which countries' products are traded. E) In the long run, national price levels play no role in determining interest rates and the relative prices at which countries' products are traded.

D

Which of the following statements is the MOST accurate? A) The law of one price does fare well in all recent studies. B) The law of one price does fare well in many recent studies. C) The law of one price sometimes fares well in recent studies. D) The law of one price does not fare well in recent studies. E) The law of one price has not been studied recently.

D

Which of the following statements is the MOST accurate? In general, under the monetary approach to the exchange rate A) the interest rate is not independent of the money supply growth rate in the short run. B) the interest rate is independent of the money supply growth rate in the long run. C) the interest rate is not independent of the money supply growth rate in the long run, but independent in the short run. D) the interest rate is not independent of the money supply growth rate in the long run. E) the interest rate is a factor of the money supply growth rate only in the short term.

D

Which of the trade rounds was the first to achieve​ across-the-board tariff​ reduction? A. The Uruguay Round. B. The Tokyo Round. C. The Kyoto Agreement. D. The Kennedy Round.

D

Which one of the following statements is the MOST accurate? A) A decrease in the money supply lowers the interest rate while an increase in the money supply raises the interest rate, given the price level and output. B) An increase in the money supply lowers the interest rate while a fall in the money supply raises the interest rate, given the price level. C) An increase in the money supply lowers the interest rate while a fall in the money supply raises the interest rate, given the output level. D) An increase in the money supply lowers the interest rate while a fall in the money supply raises the interest rate, given the price level and output. E) An increase in the money supply does not usually affect the interest rate.

D

Which one of the following statements is the MOST accurate? A) A rise in the average value of transactions carried out by a household or a firm causes its demand for money to fall. B) A reduction in the average value of transactions carried out by a household or a firm causes its demand for money to rise. C) A rise in the average value of transactions carried out by a household or a firm causes its demand for money to rise. D) A rise in the average value of transactions carried out by a household or a firm causes its demand for real money to rise. E) a decrease in the average value of transactions carried out by a household or a firm causes its demand for real money to rise.

D

Why do people argue that WTO undermines national​ sovereignty? A. WTO increases trade that leads to cultural homogenization B. WTO imposes restrictions on trade policy that are too stringent C. WTO dictates to the countries what to produce D. WTO monitors some domestic policies that affect trade.

D

8) Given P$ and Y$ A) An increase in the European money supply causes the euro to appreciate against the dollar, but it does not disturb the U.S. money market equilibrium. B) An increase in the European money supply causes the euro to appreciate against the dollar, and it creates excess demand for dollars in the U.S. money market. C) An increase in the European money supply causes the euro to depreciate against the dollar, and it creates excess demand for dollars in the U.S. money market. D) An increase in the European money supply causes the euro to depreciate against the dollar, but it does not disturb the U.S. money market equilibrium. E) An increase in the European money supply causes the euro to depreciate against the dollar, and disturbing the U.S. money market equilibrium.

D) An increase in the European money supply causes the euro to depreciate against the dollar, but it does not disturb the U.S. money market equilibrium.

29) Which of the following statements is the MOST accurate? A) In the long run, national price levels play a minor role in determining both interest rates and the relative prices at which countries' products are traded. B) In the long run, national price levels play a key role only in determining interest rates. C) In the long run, national price levels play a key role only in determining the relative prices at which countries' products are traded. D) In the long run, national price levels play a key role in determining both interest rates and the relative prices at which countries' products are traded. E) In the long run, national price levels play no role in determining interest rates and the relative prices at which countries' products are traded.

D) In the long run, national price levels play a key role in determining both interest rates and the relative prices at which countries' products are traded.

Government purchases are defined as A) only goods purchased by federal, state, or local governments. B) all goods and services purchased by the federal government. C) all goods and services purchased by the federal or state government. D) all goods and services purchased by the federal, state, or local government. E) goods and services purchased from the government.

D) all goods and services purchased by the federal, state, or local government.

An open economy A) can save only by building up its capital stock. B) can save only by acquiring foreign wealth. C) cannot save either by building up its capital stock or by acquiring foreign wealth. D) can save either by building up its capital stock or by acquiring foreign wealth. E) can save by avoiding excessive imports.

D) can save either by building up its capital stock or by acquiring foreign wealth.

An example of how GNP accounts for services provided by foreign-owned capital (and GDP does not) is A) earnings of a Spanish factory with British owners counts only in Spain's GDP. B) earnings of a Spanish factory with British owners counts only in Britain's GNP. C) earnings of a Spanish factory counts in Spain's GNP but are part of Britain's GDP. D) earnings of a Spanish factory counts in Spain's GDP but are part of Britain's GNP. E) earnings of a Spanish factory counts in Spain's GNP but not in Britain's GDP or GNP.

D) earnings of a Spanish factory counts in Spain's GDP but are part of Britain's GNP.

The Mexican government sells pesos to the United States Treasury and buys dollars.

Debit in the financial account

GNP equals GDP A) minus net receipts of factor income from the rest of the world. B) plus receipts of factor income from the rest of the world. C) minus receipts of factor income from the rest of the world. D) plus net receipts of factor income from the rest of the world. E) minus depreciation.

D) plus net receipts of factor income from the rest of the world.

A Japanese firm in Tennessee buys car parts from a subsidiary in Malaysia.

Debit int the current account

If the Chinese monetary authorities allow their currency​ (the Rembini​ / Yuan) to​ float, would you expect the dollar to appreciate or depreciate relative to the​ Yuan?

Depreciate

An appreciation of a country's currency A) decreases the relative price of its exports and lowers the relative price of its imports. B) raises the relative price of its exports and raises the relative price of its imports. C) lowers the relative price of its exports and raises the relative price of its imports. D) raises the relative price of its exports and lowers the relative price of its imports. E) raises the relative price of its exports and does not affect the relative price of its imports.

D) raises the relative price of its exports and lowers the relative price of its imports.

Nowadays, some people recommend restrictions on imports from China​ (and other​ countries) to reduce the American current account deficit. How would higher U.S. barriers to imports affect private​ saving, domestic​ investment, and government​ deficit? A. It will reduce government budget deficit. B. It will certainly reduce private savings. C. It will increase investment. D. It is impossible to tell without a general equilibrium model. Do you agree that import restrictions would necessarily reduce a U.S. current account​ deficit? A. ​No, because we cannot tell what general equilibrium effects will be. B. ​Yes, because it will reduce imports.

D, A

Which of the following is most plausible as an explanation for relative PPP holding better in the long run than in the short​ run? A. prices tend to be less sticky in the long​ run, thus lessening any deviation from PPP. B. in the short run exchange rate fluctuations may be seen as temporary by trading firms. C. it takes time for international trading firms to acquire​ and/or expand their​ "presence" in higher price markets. D. All of the above are plausible. E. A and​ B, but not C.

D. All of the above are plausible.

The fundamental reason why trade potentially benefits a country is that it A. promotes restoration of natural resources. B. increases dependency on foreign countries. C. guarantees that everyone is better off. D. expends the economy's choices.

D. expends the economy's choices.

Within each country that opens to international trade, A. workers always lose while corporations always win. B. there is a clearcut gain for all factors owners. C. there will be losses to the country's abundant factor. D. some factor owners gain, but other factors owners lose.

D. some factor owners gain, but other factors owners lose.

Assume a specific factors economy produces two goods, cloth and food, and that when representing this economy graphically, cloth is on the x-axis and food is on the y-axis. For a trading economy, A. the slope of the budget constraint is (Pc/Pf) B. the slope of the budget constraint is -(Pf/Pc) C. the budget constraint intersects the PPF at the chosen production point. D. the budget constraint is tangent to the PPF at the chosen production point.

D. the budget constraint is tangent to the PPF at the chosen production point.

A migrant worker in California sends​ $500 home to his village in Mexico.

Debit in the current account

An American church donates five tons of rice to the Sudan to help with famine relief.

Debit in the current account

An American retired couple flies from Seattle to Tokyo on Japan Airlines.

Debit in the current account

An American mutual fund manager uses the deposits of his fund investors to buy Brazilian telecommunication stocks.

Debit in the financial account

A sustained change in the monetary growth rate will A) immediately affect equilibrium real money balances by raising the money interest rate. B) eventually affect equilibrium nominal money balances by raising the money interest rate. C) eventually affect equilibrium real money balances by reducing the money interest rate. D) eventually affect equilibrium real money balances by raising the real interest rate. E) eventually affect equilibrium real money balances by raising the money interest rate.

E

An American buys a Japanese car, paying by writing a check on an account with a bank in New York. How would this be accounted for in the balance of payments? A. current account, a U.S. good import B. financial account, a U.S. asset import C. current account, a Japanese good import D. financial account, a U.S. asset export E. a current account as a U.S. good import and a financial account, a U.S. asset export.

E

An increase in A) nominal output raises the interest rate while a fall in real output lowers the interest rate, given the price level and the money supply. B) real output decreases the interest rate while a fall in real output increases the interest rate, given the price level. C) real output raises the interest rate while a fall in real output lowers the interest rate, given the money supply. D) nominal output raises the interest rate while a fall in real output lowers the interest rate, given the price level. E) real output raises the interest rate while a fall in real output lowers the interest rate, given the price level and the money supply.

E

A​ country's gross national product​ (GNP) is A. the value of all final goods and services produced by its factors of production and sold on the market. B. the value of all intermediate goods and services produced by its factors of production and sold on the market in a given time period. C. the value of all final goods produced by its factors of production and sold on the market in a given time period. D. the value of all final goods and services produced by its factors of​ production, excluding​ land, and sold on the market in a given time period. E. the value of all final goods and services produced by its factors of production and sold on the market in a given time period.

E

Because of its politically sensitive​ nature, the last economic sector to be liberalized via multilateral trade negotiations has been A. financial services B. transportation C. manufacturing D. energy E. agriculture.

E

For a given level of A) nominal GNP, changes in interest rates cause movements along the L(R,Y) schedule. B) real GNP, changes in interest rates cause a decrease of the L(R,Y) schedule. C) real GNP, changes in interest rates cause an increase of the L(R,Y) schedule. D) nominal GNP, changes in interest rates cause an increase in the L(R,Y) schedule. E) real GNP, changes in interest rates cause movements along the L(R,Y) schedule.

E

In order for the condition E$/HK$ = PUS/PHK to hold, what assumptions does the principle of purchasing power parity make? A) Only that there are no transportation costs and restrictions on trade. B) Only that the markets are perfectly competitive, i.e., P = MC. C) The factors of production are identical between countries. D) No arbitrage exists. E) HK and the US are perfectly competitive and there are no transportation costs or restrictions on trade.

E

In the United States over the past sixty years, the fraction of GNP devoted to consumption has fluctuated in a range of about A. 42 to 49 percent. B. 22 to 29 percent. C. 82 to 89 percent. D. 32 to 39 percent. E. 62 to 70 percent.

E

Interest rate differences between countries depend on A) differences in expected inflation, but not on expected changes in the real exchange rate. B) differences in expected changes in the real exchange rate, but not on expected inflation. C) neither differences in expected inflation, nor on expected changes in the real exchange rate. D) differences in expected inflation and nothing else. E) differences in expected inflation, and on expected changes in the real exchange rate.

E

Spencer and Brander's model highlights the conventional assumption that A) government involvement in business or in the economy tends to fail. B) government subsidies tend to waste taxpayer's money. C) government subsidies cannot create a successfully competing export. D) government tends to distort when it displaces Adam Smith's Invisible Hand. E) government subsidies can produce profits that exceed the subsidy's value.

E

The General Agreement on Tariffs and Trade and the World Trade Organization have resulted in A) termination of export subsidies applied to manufactured goods. B) termination of import tariffs applied to manufactures. C) termination of import tariffs applied to agricultural commodities. D) termination of international theft of copyrights. E) a number of rounds of multilateral trade agreements.

E

The PPP theory fails in reality for all of the following reasons EXCEPT A) transport costs. B) monopolistic or oligopolistic practices in goods markets. C) the inflation data reported in different countries are based on different commodity baskets. D) restrictions on trade. E) inflation rates are unrelated to money supply growth.

E

The Ricardian model of comparative advantage lends support to the argument that A) trade tends to worsen the conditions of unskilled labor in rich countries. B) trade tends to worsen the conditions of owners of capital in rich countries. C) trade tends to worsen the conditions of workers in poor countries. D) trade tends to worsen the conditions of workers in rich countries. E) trade is mutually beneficial to the countries that engage in it.

E

The United States issues a $10,000 debt forgiveness to Argentina. How is this accounted for in the balance of payments? A. current account, Argentina transfer payment B. current account, Argentina good import C. current account, U.S. service export D. financial account, U.S. asset import E. capital account, U.S. transfer payment

E

The best economic case one can make for an active industrial policy involves A) the national security argument. B) the technological spillover argument. C) the environment preservation argument. D) the high value added argument. E) raising the national income.

E

The money supply schedule is A) horizontal because MS is set by the central bank while P is taken as given. B) horizontal because MS is set by the central bank. C) vertical because MS is set by the households and firms while P is taken as given. D) vertical because MS and P are set by the central bank. E) vertical because MS is set by the central bank while P is taken as given.

E

Under sticky prices A) an interest rate rise is associated with lower expected deflation and a long-run currency appreciation, so the currency appreciates immediately. B) an interest rate rise is associated with higher expected inflation and a long-run currency appreciation, so the currency appreciates immediately. C) an interest rate rise is associated with lower expected inflation and a long-run currency depreciation, so the currency appreciates immediately. D) an interest rate rise is associated with lower expected inflation and a long-run currency depreciation, so the currency depreciates immediately. E) an interest rate rise is associated with lower expected inflation and a long-run currency appreciation, so the currency appreciates immediately.

E

Under the monetary approach to the exchange rate A) a reduction in the money supply will cause immediate currency depreciation. B) a rise in the money supply will cause currency depreciation. C) a rise in the money supply will cause immediate currency appreciation. D) a rise in the money supply will cause depreciation. E) a rise in the money supply will cause immediate currency depreciation.

E

Under the monetary approach to the exchange rate A) an interest rate decrease is associated with higher expected inflation and a currency that will be weaker on all future dates. B) an interest rate increase is associated with higher expected deflation and a currency that will be weaker on all future dates. C) an interest rate increase is associated with higher expected inflation and a currency that will be strengthened on all future dates. D) an interest rate increase is associated with higher expected deflation and a currency that will be strengthened on all future dates. E) an interest rate increase is associated with higher expected inflation and a currency that will be weaker on all future dates.

E

When all variables start out at their long-run equilibrium levels, the most important determinant of long-run swings in nominal exchange rates is A) a shift in relative money supply levels. B) a shift in relative money supply growth rates. C) a change in relative output demand. D) a change in relative output supply. E) a change in relative inflation rates.

E

When one applies the Heckscherminus−Ohlin model of trade to the issue of trademinus−related income​ redistributions, one must conclude that North South​ trade, such as U.S.minus−Mexico ​trade, A. is likely to help highly skilled workers in Mexico. B. is likely to hurt highminus−skilled workers in the U.S. C. is likely to hurt lowminus−skilled workers in Mexico. D. must help low skill workers on both sides of the border. E. is likely to hurt lowminus−skilled workers in the U.S.

E

When the U.S. placed tariffs on French wine, France placed high tariffs on U.S. chickens. This is an example of A) deadweight losses. B) multilateral negotiations. C) bilateral trade negotiations. D) international market failures. E) a trade war.

E

When the WTO met in Seattle to initiate a further move towards free international trade, thousands of activists met A) in order to promote the WTO's goals of "Trade-not Aid." B) in order to laud the WTO policy orientation which would bust local monopolies and therefore help ordinary relatively poor consumers everywhere. C) in order to laud the WTO policy of disallowing government sweetheart deals, which typically meant that corrupt governments subsidized their in-laws' conglomerates on the backs of poor taxpayers. D) in order to support the WTO efforts of bringing about a universal shift of resources in poor countries to higher efficiency and productivity uses, which would raise the real incomes of everyone. E) in order to protest WTO free trade policies that they believed hurt workers.

E

When the domestic money prices of goods are held constant A) a nominal dollar appreciation makes U.S. goods cheaper compared with foreign goods. B) a nominal dollar depreciation makes U.S. goods less appealing in foreign markets. C) a nominal dollar appreciation does not affect the prices of U.S. goods. D) a nominal dollar depreciation makes U.S. goods more expensive compared with foreign goods. E) a nominal dollar depreciation makes U.S. goods cheaper compared with foreign goods and a nominal dollar appreciation makes U.S. goods more expensive compared with foreign goods.

E

Which of the following is false about private savings and government savings? A. SP = Y - T - C B. Total savings (S) = SP + Sg. C. Unlike private saving decisions, government saving decisions are often made with an eye toward their effect on output and employment. D. The national income identity can help us to analyze the channels through which government saving decisions influence macroeconomic conditions. E. None of the above; all statements are true.

E

Which of the following statements is the MOST accurate? A) The law of one price applies only to the general price level. B) The law of one price applies to the general price level while PPP applies to individual commodities. C) The law of one price applies to individual commodities while PPP applies to both the general price level and to individual commodities. D) PPP applies only to individual commodities. E) The law of one price applies to individual commodities while PPP applies to the general price level.

E

Which one of the following statements is the MOST accurate? A) Departures from PPP are similar in both the short run and long run. B) Departures from PPP are even greater in the long run than in the long run. C) Departures from PPP are always greater in the short run than in the long run. D) It is hard to tell whether departures from PPP are greater in the short run than in the long run. E) Departures from PPP may often be greater in the short run than in the long run.

E

Which one of the following statements is the MOST accurate? A) Given PUS, when the money supply rises, the dollar interest rate declines and the dollar depreciates against the euro. B) Given YUS, when the money supply rises, the dollar interest rate declines and the dollar depreciates against the euro. C) Given PUS and YUS, when the money supply decreases, the dollar interest rate declines and the dollar depreciates against the euro. D) Given PUS and YUS, when the money supply rises, the dollar interest rate declines and the dollar appreciates against the euro. E) Given PUS and YUS, when the money supply rises, the dollar interest rate declines and the dollar depreciates against the euro.

E

The United States began to report its gross domestic product (GDP) only since A) 1900. B) 1921. C) 1931. D) 1941. E) 1991.

E) 1991.

What is the exchange rate between the dollar and the British pound if a pair of American jeans costs 60 dollars in New York and 30 Pounds in London? A) 1.5 dollars per British pound B) 0.5 dollars per British pound C) 2.5 dollars per British pound D) 3.5 dollars per British pound E) 2 dollars per British pound

E) 2 dollars per British pound

In 2010, about A) 20 percent of foreign exchange transactions involved exchanges of foreign currencies for U.S. dollars. B) 10 percent of foreign exchange transactions involved exchanges of foreign currencies for U.S. dollars. C) 30 percent of foreign exchange transactions involved exchanges of foreign currencies for U.S. dollars. D) 40 percent of foreign exchange transactions involved exchanges of foreign currencies for U.S. dollars. E) 85 percent of foreign exchange transactions involved exchanges of foreign currencies for U.S. dollars.

E) 85 percent of foreign exchange transactions involved exchanges of foreign currencies for U.S. dollars.

Which of the following is TRUE? A) A country with a current account surplus is earning more from its exports than it spends on imports. B) A country could finance a current account deficit by using previously accumulated foreign wealth to pay for its imports. C) A country with a current account deficit must be increasing its net foreign debts by the amount of the deficit. D) We can describe the current account surplus as the difference between income and absorption. E) All of the above are true of current account balances.

E) All of the above are true of current account balances.

7) Which one of the following statements is the MOST accurate? A) Given P$, when the money supply rises, the dollar interest rate declines and the dollar depreciates against the euro. B) Given Y$, when the money supply rises, the dollar interest rate declines and the dollar depreciates against the euro. C) Given P$ and Y$, when the money supply decreases, the dollar interest rate declines and the dollar depreciates against the euro. D) Given P$ and Y$, when the money supply rises, the dollar interest rate declines and the dollar appreciates against the euro. E) Given P$ and Y$, when the money supply rises, the dollar interest rate declines and the dollar depreciates against the euro.

E) Given P$ and Y$, when the money supply rises, the dollar interest rate declines and the dollar depreciates against the euro.

Which of the following is FALSE about private savings and government savings? A) SP = Y - T - C B) Unlike private saving decisions, government saving decisions are often made with an eye toward their effect on output and employment. C) Total savings (S) = SP + . D) The national income identity can help us to analyze the channels through which government saving decisions influence macroeconomic conditions. E) None of the above; all statements are true.

E) None of the above; all statements are true.

Net unilateral transfers A) are part of a national income. B) are part of a country's product. C) must be added to NNP in calculations of national income. D) are part of a country's GNP. E) Only A and C.

E) Only A and C.

Which one of the following statements is FALSE? A) The United States had accumulated substantial foreign wealth by the early 1980s. B) The 1980s witnessed a sustained current account deficit of proportions unprecedented in the twentieth century opened up. C) In 1987, the country became a net debtor to foreigners for the first time since World War I. D) U.S. foreign debt has continued to grow and now stands at 25 percent of GNP. E) The U.S. foreign debt was paid off in the 1990s, allowing the U.S. to attain a current account surplus. However, the deficit has returned in recent years.

E) The U.S. foreign debt was paid off in the 1990s, allowing the U.S. to attain a current account surplus. However, the deficit has returned in recent years.

25) Under the monetary approach to the exchange rate A) a reduction in the money supply will cause immediate currency depreciation. B) a rise in the money supply will cause currency depreciation. C) a rise in the money supply will cause immediate currency appreciation. D) a rise in the money supply will cause depreciation. E) a rise in the money supply will cause immediate currency depreciation.

E) a rise in the money supply will cause immediate currency depreciation.

Finding interest rates:

Future rate (Et+1)/Spot (Et)=1+domestic ir/1+foreign *multiply the foreign ir over to the other side.. => domestic ir=[(foreign)(future)/spot]-1 =%

22) The PPP theory fails in reality for all of the following reasons EXCEPT A) transport costs. B) monopolistic or oligopolistic practices in goods markets. C) the inflation data reported in different countries are based on different commodity baskets. D) restrictions on trade. E) inflation rates are unrelated to money supply growth.

E) inflation rates are unrelated to money supply growth.

National income equals GNP A) less depreciation, less net unilateral transfers, less indirect business taxes. B) less depreciation, plus net unilateral transfers, plus indirect business taxes. C) less depreciation, less net unilateral transfers, plus indirect business taxes. D) plus depreciation, plus net unilateral transfers, less indirect business taxes. E) less depreciation, plus net unilateral transfers, less indirect business taxes.

E) less depreciation, plus net unilateral transfers, less indirect business taxes.

Disposable income is National income A) less taxes collected from households and firms by the government. B) plus net taxes collected from households and firms by the government. C) less net taxes collected from firms by the government. D) less net taxes collected from households by the government. E) less net taxes collected from households and firms by the government.

E) less net taxes collected from households and firms by the government.

5) For a given level of A) nominal GNP, changes in interest rates cause movements along the L(R,Y) schedule. B) real GNP, changes in interest rates cause a decrease of the L(R,Y) schedule. C) real GNP, changes in interest rates cause an increase of the L(R,Y) schedule. D) nominal GNP, changes in interest rates cause an increase in the L(R,Y) schedule. E) real GNP, changes in interest rates cause movements along the L(R,Y) schedule.

E) real GNP, changes in interest rates cause movements along the L(R,Y) schedule.

2) An increase in A) nominal output raises the interest rate while a fall in real output lowers the interest rate, given the price level and the money supply. B) real output decreases the interest rate while a fall in real output increases the interest rate, given the price level. C) real output raises the interest rate while a fall in real output lowers the interest rate, given the money supply. D) nominal output raises the interest rate while a fall in real output lowers the interest rate, given the price level. E) real output raises the interest rate while a fall in real output lowers the interest rate, given the price level and the money supply.

E) real output raises the interest rate while a fall in real output lowers the interest rate, given the price level and the money supply.

27) Under a flexible-price monetary approach to the exchange rate A) when the domestic money supply falls, the price level would eventually fall, increasing the interest rate. B) when the domestic money supply falls, the price level would fall right away, causing a reduction in the interest rate. C) when the domestic money supply falls, the price level would fall right away, causing an increase in the interest rate. D) when the domestic money supply falls, the price level would eventually fall, keeping the interest rate constant. E) when the domestic money supply falls, the price level would fall right away, keeping the interest rate constant.

E) when the domestic money supply falls, the price level would fall right away, keeping the interest rate constant.

Expected real interest rates in all countries will be identical only when A. relative PPP is expected to hold. B. inflation rates are identical in all countries. C. real exchange rates are not expected to change. D. All of the above. E. A and C but not B.

E. A and C but not B.

requiring home country standards

Eliminates the fear of a​ race-to-the-bottom and shifts the costs of improved standards to firms and consumers in the​ high-income countries.​ However, it only addresses the problem of firms in​ high-standard countries that go abroad to potentially​ low-standard countries, and not vice versa.

Debt that is money owed to​ non-residents and must be paid in foreign currency is known as

External debt

It is not possible for government deficits to decline while increasing the current account deficit because economic variables have to be in equilibrium.

False

In theory the​ equality: current account​ + capital account​ + financial account​ = 0 must hold. In reality a statistical discrepancy is often included to achieve this balance. Which account is the likely culprit of this​ discrepancy?

Financial account

Accounts in the balance of payments

Financial, capital, and current account

Suppose there is a permanent reduction in aggregate real money​ demand, that​ is, a negative shift in the aggregate real money demand function. Trace the​ short-run and​ long-run effects on the exchange​ rate, interest​ rate, and price level. In the long​ run, the exchange rate will

In the short​ run, the interest rate will​ fall, the expected return on foreign currency will​ increase, which results in a depreciation of the domestic currency. In the long​ run, the price level will rise to equate money demand with money supply at the initial​ (long run) interest rate. appreciate some as the interest rate returns to its​ long-run value but will depreciate relative to its initial value.

Which of the following represents direct foreign​ investment?

Intel moves part of its production to a plant in Malasya

In the current​ Post-Industrial economy, international trade in services​ (including banking and financial​ services):

Is relatively small

Private saving can be given by the following​ equation: Sp=I+CA+(G−T). This equation can be rewritten​ as: CA = Sp−I+(T−G). ​Nowadays, some people recommend restrictions on imports from China​ (and other​ countries) to reduce the American current account deficit. How would higher U.S. barriers to imports affect private​ saving, domestic​ investment, and government​ deficit? Do you agree that import restrictions would necessarily reduce a U.S. current account​ deficit?

It is impossible to tell without a general equilibrium model. No, because we cannot tell what general equilibrium effects will be.

LOOP vs PPP

LOOP looks at the price of one good whereas PPP looks at the price of a basket of goods

increasing international negotiations

Leaves it up to nations to collectively decide on their own methods of​ enforcement, but may put at a disadvantage those nations with little influence or bargaining power and may also lead to excessive bureaucratization.

The largest trading of foreign exchange occurs in

London

Which of the Federal​ Reserve's measures of the monetary aggregates is composed of the most liquid​ assets? Among M1 and​ M2, which is the largest​ measure?

M1 M2

Which of the following functions corresponds to a liquidity preference function and correctly identifies the relationship between the​ left-hand side variables and the​ right-hand side​ variables? ​ (Note that Md is the quantity of money​ demanded, P the price​ level, i interest​ rates, Y real ​income, and f is the function operator that relates inputs to an​ output.)

M^d/P = f ( i (-), Y (+))

A current account deficit

May be beneficial to countries, since the country might benefit from technology transfer

The aggregate demand for money can be expressed by:

Md = P × L(R,Y).

What is the​ short-run effect on the exchange rate of an increase in domestic real​ GNP, given expectations about future exchange​ rates?

Money demand​ increases, the domestic interest rate​ increases, and the domestic currency appreciates.

Calculating the future value of an investment if it is invested in the US:

Multiply the initial investment by (1+the interest rate)

GNP​ (Gross National​ Product) equals GDP plus

Net recipients of factor income from the rest of the world and net unilateral transfers

Do data on the U.S. official settlements balance give an accurate picture of the extent to which foreign central banks buy and sell dollars in currency​ markets?

No, this account provides only a partial picture because it shows a net value of all transactions.

When a government​ (typically corrupt) incurs debt without the consent of the people and the debt is not used for the​ people's benefit, the debt is

Odious

Which of the following is not a difference between the portfolio capital flows and investment capital flows?

Portfolio capital flows are a net increase in the amount of resources available for investment

What are the determinants of the aggregate money​ demand?

Price​ level, national​ income, and interest rate.

Which equation correctly shows how budget deficits of a country are linked to its current account​ balance?

Private savings + Gov savings = Investment + current account

Which of the following is not part of the definition for Gross National​ Product?

Produced within a country's borders

Refer to the model of the long run real exchange rate to the right. Using the line drawing tool​, show the effect of an increase in the demand for U.S. output. Properly label your line. Carefully follow the instructions above and only draw the required object.

RD1 is below RD

_ Trade Agreements: _ _ Area = -free trade among member countries -each country free to choose _ _ to outside _ -ex: _ _ _ = -free trade among member countries -common _ _ to outside countries -ex: _ _ _ = -customs union -free _ of labor and _ among member countries -ex: _ (goes beyond with monetary union)

Regional; Free Trade; tariff rate; countries; NAFTA; Customs Union; tariff rate; Mercosur; Common Market; movement; capital; EU

Suppose that Europeans decide to purchase fewer US goods for a given relative price of US and European goods. This change will affect the _______ of US goods versus European goods.

Relative Demand

Interest rate differences between countries depend exclusively upon differences in expected inflation only when

Relative PPP holds.

Expected real interest rates in all countries will be identical only when

Relative PPP is expected to hold. Real exchange rates are not expected to change.

Explain risk and liquidity

Risk is the variability an asset contributes to a saver's wealth. An asset's real return can be unpredictable and savers dislike this uncertainty if the return fluctuates widely. Liquidity refers to the ease with which an asset can be sold or exchanged for goods. Cash is the most liquid of assets because it is always acceptable at face value as payment for goods or other assets. Thus, savers consider an asset's liquidity and its expected return and risk in deciding how much of it to hold.

Adjusting:

Say that the dollar is over valued in comparison to the pound, that may lead to American consumers purchasing more in pounds and european goods (bc they can get more for their money) this would lead to an increase in the $ money supply and and increase in the demand of the pound. This would mean the dollar would be depreciating and the pound would appreciate in relation to each other resetting the exchange rate (adjusting) to the PPP. (not reality though)

The Law of One Price (LOOP)

Simply says that the same good in different competitive markets must sell for the same price, when transportation costs and barriers between those markets are not important.

In our discussion of​ short-run exchange rate​ overshooting, we assumed that real output was given. Assume instead that an increase in the money supply raises real output in the short run​ (an assumption that will be justified in Chapter​ 16). How does this affect the extent to which the exchange rate overshoots when the money supply first​ increases? Is it likely that the exchange rate​ undershoots?

Since the increase in output will increase the demand for​ money, the interest rate will not fall as​ much; thus, the overshoot will be smaller. The only way for the exchange rate to undershoot is if the interest rate rises when the money supply increases.

Which economic institution determines or controls the money supply in the​ U.S.?

The Federal Reserve.

What is the interest parity condition?

The condition that the expected returns on deposits of any two currencies are equal when measured in the same currency is called the interest parity condition. It implies that potential holders of foreign currency deposits view them as equally desirable assets, i.e. risk assumed away.

What is TRUE about a vehicle currency?

The dollar is sometimes called a vehicle currency because of its pivotal role in many foreign exchange deals.

In​ theory, the​ equality: current account​ + capital account​ + financial account​ = 0 must hold. In reality a statistical discrepancy is often included to achieve this balance. Which account is the likely culprit of this​ discrepancy?

The financial account

The dollar rate of return on euro deposits is

approximately the euro interest rate plus the rate of depreciation of the dollar against the euro

Explain why the interest parity condition must hold if the foreign exchange market is in equilibrium:

The foreign exchange market is in equilibrium when deposits of all currencies offer the same expected rate of return. Potential holders of foreign currency deposits view them all as equally desirable assets. If expected rate of return on any currency deposit is higher or lower than the other, there will exist an excess supply or demand for that currency because one will yield a higher return than the other.

We know that the U.S. is a substantial net debtor to foreigners.​ How, then, is it possible that the U.S. received more foreign asset income than it paid​ out?

The rate of return on U.S. holdings of foreign assets is higher than the rate foreigners earn on U.S. assets.

If a currency reform has no effects on the​ economy's real​ variables, why do governments typically institute currency reforms in connection with broader programs aimed at halting runaway​ inflation? ​ (There are many instances in addition to the Turkish case mentioned in the text. Other examples include​ Israel's switch from the pound to the​ shekel, Argentina's switches from the peso to the austral and back to the​ peso, and​ Brazil's switches from the cruzeiro to the​ cruzado, from the cruzado to the​ cruzeiro, from the cruzeiro to the cruzeiro​ real, and from the cruzeiro real to the​ real, the current​ currency, which was introduced in​ 1994.)

There may be a psychological benefit in that currency reform can have a positive effect on inflation expectations. ​ However, for the stabilization plan to​ succeed, it must be backed up by concrete policies to reduce monetary growth.

How would you expect a fall in a​ country's population to alter its aggregate money demand​ function? Would it matter if the fall in population were due to a fall in the number of households or to a fall in the average size of a​ household?

There would be a decrease in money demand because there would be fewer transactions. The decrease in money demand would be larger if the decrease in population was due to a fall in the number of households.

Suppose the denominator increases

This would appreciate the dollar because it would cause the exchange rate to fall. You would be dividing by a higher denominator cause the overall number to be smaller.

The velocity of​ money, V, is defined as the ratio of real GNP to real money​ holdings, Upper V equals Upper Y divided by left parenthesis Upper M divided by Upper P right parenthesis in this​ chapter's notation. Use Upper M Superscript s Baseline divided by Upper P equals Upper L left parenthesis Upper R comma Upper Y right parenthesis to derive an expression for velocity. Explain how velocity varies with changes in R and in Y. ​ (Hint: The effect of output changes on V depends on the elasticity of aggregate money demand with respect to real​ output, which economists believe to be less than​ unity.) What is the relationship between velocity and the exchange​ rate?

V=Y/L(R,Y) An increase in either R or Y will increase velocity. There is a positive relationship. ​ Thus, an increase in velocity is associated with an appreciation of the exchange rate.

The Japanese currency is called the

Yen

Your country has a positive International Investment Position​ (IIP) with another country. This statement means

Your country's citizens own more assets in the other country than the other country's citizens own in your country

Capital gains and losses on a​ country's net foreign assets are not included in the national income measure of the current account. How would economic statisticians have to modify the national income identity Y = C+I +G + X − M if they did wish to include such gains and losses as part of the definition of the current​ account?

Y​ = C​ + I​ + G​ + X − M​ + Net capital gain.

Which of the following DOES NOT help to reduce the frequency of child​ labor?

a country's movement away from manufacturing and towards more agricultural production

Low wages and poor working conditions in many U.S. trade partners

are facts emphasized by U.S. labor in its contract negotiations.

For a given euro interest rate and constant expected exchange rate,

a rise in the interest rate offered by dollar deposits causes the dollar to appreciate

Money serves as all of the following except

a symbol that is made of or can be redeemed for a fixed amount of precious metal.

​Large-scale wars typically bring a suspension of international trading and financial activities. Exchange rates lose much of their relevance under these​ conditions, but once the war is over governments wishing to fix exchange rates face the problem of deciding what the new rates should be. The PPP theory has often been applied to this problem of postwar exchange rate realignment. If you were the British Chancellor of the Exchequer and World War I had just​ ended, a good first approximization to identifying the appropriate​ post-war dollar/pound exchange rate might be obtained by A. adjusting the​ pre-war rate according to the contribution each country made to the war effort. B. adjusting the​ pre-war rate according to the income changes experienced by the two countries during the war. C. allowing market forces to determine the appropriate rate. D. adjusting the​ pre-war rate according to the price level changes experienced by the two countries during the war. E. simply restoring the​ pre-war rate. ​If, during the​ war, the U.S. price level increased by 15 ​% while the U.K. level rose by 10 ​%, a​ best-guess post-war​ dollar/pound exchange rate would​ be, compared to the​ pre-war rate,?? Using the PPP theory in this manner may be a bad idea if A. relative demands for the goods produced in the countries changed substantially. B. the two countries had significantly different wartime changes in productive capacity. C. productivity trends in the two countries were disparate during the war. D. all of the above are reasons why reliance upon PPP theory may be less than satisfactory.

adjusting the​ pre-war rate according to the price level changes experienced by the two countries during the war. 5% higher all of the above are reasons why reliance upon PPP theory may be less than satisfactory.

Harmonization of standards implies that countries decide to

adopt a common set of standards in an area of concern, such as product safety or labor

Individuals base their demand for an asset on

all of these.

a pollution haven is said to exist when

an economic activity becomes concentrated in countries or regions having less strict environmental controls than elsewhere.

If the dollar interest rate is 10 percent, the euro interest rate is 6 percent, and the expected dollar depreciation against the euro is 4 percent, then

an investor should be indifferent between dollars and euros

If the dollar interest rate is 10 percent, the euro interest rate is 12 percent, then

an investor should invest only in dollars if the expected dollar appreciation against the euro is 4 percent

If the dollar interest rate is 10 percent, the euro interest rate is 6 percent and if the expected dollar depreciation against the euro is 8 percent, then

an investor should invest only in euros

WTO functions: -lower trade _ -resolve _ issues -administering WTO trade _ -forum for trade _ -handling trade _ -monitoring national trade _ -technical assistance and training for _ countries -_ with other international organizations

barriers; dumping; agreements; negotiations; disputes; policies; developing; cooperation

What is NOT an example of a financial derivative?

bonds

in order for countries to be integrated, standards______

can differ, even persistently

A permanent increase in a country's money supply

causes a proportional increase in its price level.

For a fixed interest rate, a rise in the expected future exchange rate causes

causes a rise in the current exchange rate

What major actor is at the center of the foreign exchange market?

commercial banks

Labor and environmental activists have advanced a number of arguments in favor of using trade barriers to enforce labor and environmental standards. Which of the following is not one of these​ arguments?

countries will only improve standards if coerced through the imposition of trade barriers

What is the main problem with imposing environmental standards?

country differences in income and preferences complicates the definition of appropriate standards

In the short run of a model with sticky​ prices, an expansion in the money supply reduces the nominal interest rate and depreciates the currency. In this case the​ country's expected real interest rate will ??? . In​ addition, the real exchange​ rate, after initially ??? ​, will subsequently ??? . The latter movement of the real exchange rate satisfies the real interest parity condition which indicates that a​ country's currency will be expected to undergo a real appreciation when its real interest rate ??? real interest rates elsewhere.

decrease increasing decrease falls below

The diagram to the right shows the determination of the​ long-run real exchange rate between​ Home's currency and​ Foreign's currency, q Subscript h divided by f . Suppose that the overall level of spending​ doesn't change, but Home residents decide to spend less of their income on nontraded products and more on tradables. Using the line drawing tool​, assess the impact of this spending shift on q Subscript h divided by f . Properly label this line. Carefully follow the instructions above and only draw the required object. According to your​ graph, the Home currency has undergone a real ??? against​ Foreign's currency. Starting again with q Subscript font size decreased by 1 font size increased by 1 h divided by font size decreased by 1 font size increased by 1 f Superscript font size decreased by 1 1 ​, this time suppose that Foreign residents shift their demand away from their own goods and toward Home's exports . Using the line drawing tool​, assess the impact of this spending shift on q Subscript h divided by f . Properly label this line. Carefully follow the instructions above and only draw the required object. According to this second​ graph, the Home currency has undergone a real ??? against​ Foreign's currency.

depreciation appreciation RD2 shift right

A(n) _________ of a nation's currency will cause imports to ______________ and exports to ___________, all other things held constant.

depreciation / decrease / increase

Arguments about labor standards are confined primarily to arguments between​ high-income countries on the one​ hand, and​ low- and​ middle-income countries on the other because these countries have underlying conditions that are very

dissimilar

Why do ppl argue that WTO undermines national sovereignty? WTO monitors some _ policies that affect _

domestic; trade

Advantages of International Trade Agreements: 1. agreement mobilizes _ support for freer _ -compare the domestic _ interest groups involved -ex: mutual tariff cut -producers of _ good _ --> oppose freer trade -producers of _ good _ --> _ freer trade 2. avoids trade war -_ _ in game theory: left to each country's individual action, the only _ is both countries _ (0/0 welfare) -_'s _: countries by individual action cannot achieve the highest _ (10/10) -countries achieve a _ superior (mutually better) outcome (10/10) by _ to do free trade with each other

domestic; trade; producer; import; lose; export; support; Nash equilibrium; outcome; protecting; prisoner's dilemma; welfare; Pareto; agreeing

What obstacles to increased international economic integration begin to appear after trade barriers are​ removed?

each country may set up different standards generating conflicts over enforcement and application, a countrys domestic regulations may unintentionally limit international commerce for that country

The argument that strategic planning is not likely to be practical due to insufficient information means that

even if the government had all the relevant information in a given industry, it still could not decide whether a subsidy would enhance the public's welfare.

Spencer and Brander's model highlights the existence of

excess returns, or rents, available in non-competitive markets.

If there is initially

excess supply of money, the interest rate falls, and if there is initially an excess demand, it rises.

Relative PPP

exchange rates equal changes in prices (inflation) between two periods:

The Shipbreakers of Alang utilize much labor and little capital, thereby supporting the applicability of the

factor proportions explanation of the sources of comparative advantage.

Spencer and Brander's model highlights the conventional assumption that

government subsidies can produce profits that exceed the subsidy's value.

A permanent change in the level of the supply of money

has no effect on the long-run values of the interest rate and real output.

What type of fund caters to wealthy individuals, are not bound by government regulations, and are actively traded in foreign exchange markets?

hedge funds

Japan's protection of its semiconductor (RAM) producers is today seen as an object lesson in

how strategic planning may backfire and cause a large waste of resources.

Nondeliverable forward exchange markets in centers such as Hong Kong and Singapore help to circumvent what problem?

inconvertible currencies cannot be traded in foreign markets

A decrease in the money supply leads to an increase in the value of the U.S. dollar and a decrease in the value of foreign currency. This in turn, leads to a decrease in net exports and aggregate demand. A decrease in the money supply leads an to increase interest rates. This, in turn, leads to a decrease in investment spending by firms and aggregate demand.

increase decrease decrease increase decrease

One common critique of the WTO is that it overturns national environmental protections and forces countries to lower their standards. For​ example, when the United States tried to protect endangered sea​ turtles, the WTO prevented it. This action illustrates that

individual countries can raise their​ standards, but must do so in accordance with WTO rules.

Suppose that the one-year forward price of euros in terms of dollars is equal to $1.113 per euro. Further, assume that the spot exchange rate is $1.05 per euro, and the interest rate on dollar deposits is 10 percent and on euro it is 4 percent. Under these assumptions:

interest parity does hold

What are the main factors determining the aggregate money demand?

interest rate, the price level and real national income

If the dollar interest rate is 10 percent and the euro interest rate is 6 percent, than an investor should

invest only in dollars if the exchange rate is expected to remain constant

If the dollar interest rate is 4 percent, the euro interest rate is 6 percent, then

invest only in euros if the exchange rate is expected to remain constant

A foreign exchange swap

is a spot sale of a currency combined with a forward repurchase of the currency

If the Nominal exchange rate over values or under values the dollar:

it should adjust

A reduction in a country's money supply causes:

its currency to appreciate in the foreign exchange market.

An increase in a country's money supply causes

its currency to depreciate in the foreign exchange market while a reduction in the money supply causes its currency to appreciate.

Forward and spot exchange rates

move closely together and are equal on the value date

Labor standards are

multifaceted, covering both basic rights​ (freedom form forced​ labor) as well as civic rights​ (union representation).

The following is an example of Radio Shack hedging its foreign currency risk

needing to pay 9,000 yen per radio to its suppliers in a month, Radio Shack makes a forward-exchange deal to buy yen

The financial account balance is

net increase in foreign ownership of​ U.S.-based nonreserve assets plus the net increase in foreign ownership of​ U.S.-based reserve assets minus the net increase in the U.S.​ government's nonreserve foreign assets minus the net increase in U.S. private assets abroad minus the net increase in U.S. ownership of official reserve assets

GNP​ (Gross National​ Product) equals GDP plus

net receipts of factor income from the rest of the world.

The current account balance is

net unilateral transfers plus exports of goods and services plus investment income received in the U.S. minus imports of goods and services minus investment income paid abroad by the U.S.

In the area of labor​ standards, variation tends to be substantial. For which of the following labor standards is there no ​variation?

none of the above have universal acceptance

Regarding the arguments that have been advanced for the use of trade barriers as an enforcement​ measure, the claim that​ "pollution havens" are fostered is_______ by empirical evidence; the claim that countries with low standards achieve an unfair advantage is _______; and the claim that a "race to the bottom" occurs is _____ by the evidence

not supported. not supported, not supported

The covered interest rate parity condition can be stated as follows: The interest rate on dollar deposits equals the interest rate on euro deposits __________________ the forward ________________ on dollars against euros.

plus / discount

The covered interest rate parity condition can be stated as follows: The interest rate on dollar deposits equals the interest rate on euro deposits __________________ the forward ________________ on euros against dollars.

plus / premium

If a home country forces home firms to follow home environmental standards when home firms operate at home or in a foreign​ location

pressures for a race to the bottom will decrease

In the second half of the 1990s a rapidly growing movement focused on the harm caused by international trade to

production workers in both rich and poor countries.

The existence of positive externalities due to the impossibility of full appropriability

provides support for government protectionism.

Absolute PPP

purchasing power parity that has already been discussed. Exchange rates equal the level of relative average prices across countries.

An appreciation of a country's currency

raises the relative price of its exports and lowers the relative price of its imports

The best economic case one can make for an active industrial policy involves

raising the national income

Suppose that American consumers decide to purchase more European goods for a given relative price of US and European goods. This change will affect the ??? of US goods versus European goods. The graph on the right shows the relative supply of US versus European real output. ​1.) Using the line drawing tool​, draw a line depicting initial relative demand and label it​ "Initial." ​2.) Using the line drawing tool​, now add a line that depicts new relative​ demand, after relative demand for European goods increase s. Label it​ "New." Carefully follow the instructions above and only draw the required objects. If relative demand for European goods increase ​s, the dollar will ??? in real terms. As a result of this​ change, the dollar will also ??? in nominal terms in the​ long-run.

relative demand depreciate depreciate initial demand is below New demand

Suppose that American consumers decide to purchase fewer European goods for a given relative price of US and European goods. This change will affect the ??? of US goods versus European goods. The graph on the right shows the relative supply of US versus European real output. ​1.) Using the line drawing tool​, draw a line depicting initial relative demand and label it​ "Initial." ​2.) Using the line drawing tool​, now add a line that depicts new relative​ demand, after relative demand for European goods decrease s. Label it​ "New." Carefully follow the instructions above and only draw the required objects. If relative demand for European goods decrease ​s, the dollar will ??? in real terms. As a result of this​ change, the dollar will also ??? in nominal terms in the​ long-run.

relative demand appreciate appreciate New is below initial (demand line)

In effect, the U.S. does subsidize high-tech firms by subsidizing R&D. This is done through

relatively accelerated "depreciation" of R&D investment in the Federal tax codes.

The Shipbreakers of Alang represent a perfect example of how a developing country can apply the principles of the Heckscher-Ohlin model, since

shipbreaking is a labor-intensive operation in India, and India's availability of capital per worker is less than that of its trade partners.

The aggregate real money demand schedule L(R,Y)

slopes downward because a fall in the interest rate raises the desired real money holdings of each household and firm in the economy.

It is argued that high-tech industries typically generate new technologies but cannot fully appropriate the commercial benefits associated with their inventions or discoveries. If this is true then in order to maximize a country's real income, the government should

subsidize the high-tech firms.

The opportunity cost of money holdings is

the alternative interest income foregone from not holding some other asset.

What is NOT a major actor in the foreign exchange market?

tourists

Which of the following assets is the most​ liquid?

traveler's check

External debt is

ubiquitous, or omnipresent - all countries have it

The money supply schedule is

vertical because M s is set by the central bank while P is taken as given.

The money supply schedule is

vertical because MS is set by the central bank while P is taken as given.

Spot exchange rates and forward exchange rates are equal when

when the value date and the date of the spot transaction are the same

The figure on the right shows the United​ States' end-of-year international investment position​ (IIP) as a percent of its nominal GDP for the period​ 1976-2012. The United States has run current account deficits in almost every year since the​ mid-1980s. Do the data in the graph therefore surprise​ you?

​No, increasingly large current account deficits coupled with slowing growth in nominal GDP imply the trend shown for​ (IIP/NGDP) at right.

GNP accounts avoid double counting by including only the value of final goods and services sold on the market. Should the measure of imports and exports used in the GNP accounts therefore be defined to include only imports and exports of final goods and services received from and sold to other​ countries?

​No, total values and imports and exports should be included in the calculation of the GNP.


Related study sets

Go further with you Google Analytics Data

View Set

Ch 6: strengthen a company's competitive positions

View Set

REL 130 - Periodic Exam 2 - Fall20

View Set

CH 2 Health Care Delivery System

View Set

mastering genetics ch 10 DNA REPLICATION

View Set

Chapter 26 and 27 - Requirements Governing Licenses and Transaction Related Issues

View Set